You are on page 1of 84

IASBABA

ALL INDIA PRELIMS TEST SERIES(AIPTS)


TOTAL -50 TEST
✓ 23-SUBJECTWISE TEST ✓ 10- CSAT FULL LENGTH
12- CURRENT AFFAIRS TEST
TEST ✓ 05 - G.S FULL LENGTH
✓ 03 THEMATIC REVISION TEST
TEST

Test-10
Current Affairs - September
IASBABA’S PRELIMS TEST SERIES (AIPTS) 2024 – TEST 10 (CURRENT AFFAIRS – SEPT ’23)

Q.1) Consider the following statements d) Statement I is incorrect Statement II


about Red Sand Boa: is correct
1. It is an ovoviviparous venomous
snake. Q.3) Consider the following statements
2. It is found in arid and semi-arid about the Bharat Stage (BS) norms:
regions. 1. They are emission standards
3. It has applications in medicine and instituted by the Government of
cosmetics. India to regulate the output of air
4. It is listed as vulnerable in the IUCN pollutants from motor vehicles.
List. 2. They are based on the European
How many of the given statements are emission standards and are
correct? implemented by the Central
a) Only one Pollution Control Board in India.
b) Only two 3. Every newly sold and registered
c) Only three vehicle in India is required to
d) All four adhere to the BS-IV version of
emission regulations.

Q.2) Consider the following statements: How many of the above statements are
correct?
Statement I:
a) Only one
Africa holds the potential to emerge as a b) Only two
prominent force in renewable energy c) All three
generation. d) None
Statement II:
It boasts a solar PV technical potential of Q.4) Consider the following statements:
7,900 GW, hydropower potential of 1,753 1. In the Mitakshara school of
GW, and wind energy potential of 461 GW inheritance, the property is
along with geothermal energy potential. inherited by the successors only if
Which one of the following is correct with they were born in the family of the
respect to the above statements? property owners.
a) Both Statement I and Statement II 2. In the Dayabhaga school of
are correct and Statement II is the inheritance, the property goes to
correct explanation for Statement I the successors only when the
b) Both Statement I and Statement II property owner is dead.
are correct and Statement II is not 3. The Mitakshara school of
the correct explanation' for inheritance is mainly practiced in
Statement I Assam and West Bengal.
c) Statement I is correct but 4. The Dayabhaga school of
Statement II is incorrect inheritance is practiced in all of
IASBABA’S PRELIMS TEST SERIES (AIPTS) 2024 – TEST 10 (CURRENT AFFAIRS – SEPT ’23)

India except in West Bengal and University Grants Commission


Assam. (UGC).
How many of the given statements are 2. It enjoys the academic status and
correct? privileges of a university but cannot
a) Only one design its syllabus and courses.
b) Only two Choose the correct code:
c) Only three a) 1 only
d) All four b) 2 only
c) Both 1 and 2
d) Neither 1 nor 2
Q.5) Consider the following statements
about the National Pension System (NPS):
1. Under this system, a unique Q.7) In the context of Indian polity,
Permanent Retirement Account consider the following statements:
Number (PRAN) is generated and 1. The Constitution specifies that six
maintained by the Central months should not elapse between
Recordkeeping Agency (CRA) for two parliamentary sessions.
individual subscribers. 2. India’s Parliament has a fixed
2. It is being administered and calendar of sittings and the
regulated by the Pension Fund President determines the date and
Regulatory and Development duration of parliamentary sessions.
Authority (PFRDA) set up under the Choose the correct code:
PFRDA Act, 2013. a) 1 only
3. Under this system, a Tier-I account b) 2 only
is a pension account having c) Both 1 and 2
restricted withdrawals while Tier-II d) Neither 1 nor 2
is a voluntary account that offers
liquidity of investments and
withdrawals. Q.8) Consider the following statements
regarding Walking leaves:
How many of the above statements are
1. They are known for their unusual
correct?
appearance as they look similar to
a) Only one
parts of plants.
b) Only two
2. They feed on plants and typically
c) All three
inhabit densely vegetated areas.
d) None
Choose the correct code:
a) 1 only
Q.6) Consider the following statements b) 2 only
regarding the deemed to be university: c) Both 1 and 2
1. It is declared by the central d) Neither 1 nor 2
government on the advice of the
IASBABA’S PRELIMS TEST SERIES (AIPTS) 2024 – TEST 10 (CURRENT AFFAIRS – SEPT ’23)

Q.9) Which of the following are the reasons Q.11) Consider the following statements
for the declining population of penguins? regarding Aarogya Maitri Cube:
1. Global warming causing early sea 1. It is the world’s third disaster
ice melting in Antarctica. hospital that can be airlifted.
2. Unstable sea ice causes problems in 2. It has been developed indigenously
breeding and raising their offspring. as part of India’s Aarogya Maitri
3. The increasing krill production initiative.
because of climate change. 3. It contains essential equipment and
How many of the above statements are supplies, such as an operation
correct? theatre, etc.
a) Only one How many of the above statements are
b) Only two correct?
c) All three a) Only one
d) None b) Only two
c) All three
d) None
Q.10) Consider the following statements
regarding the DIKSHA Platform:
1. It provides quality e-content for Q.12) Consider the following statements:
school education in States/UTs. 1. Poshan 2.0 is a central sector
2. It is an initiative of the Ministry of scheme that addresses
Education for digital online malnutrition among children,
education. adolescent girls, pregnant and
3. It has embedded assistive lactating women.
technologies for learners with 2. The 6th Rashtriya Poshan Maah is
visual or hearing impairments. celebrated by the Ministry of
4. It has integrated personalized Health and Family Welfare to
adaptive learning(PAL) software to comprehensively
cater to the specific learning needs address malnutrition using a life-
of students. cycle approach.
How many of the given statements are 3. The theme of the
correct? 6th Rashtriya Poshan Maah of 2023
a) Only one is ‘Suposhit Bharat, Sakshar Bharat,
b) Only two Sashakt Bharat’.
c) Only three How many of the above statements are
d) All four correct?
a) Only one
b) Only two
c) All three
d) None
https://upscmaterial.online/

IASBABA’S PRELIMS TEST SERIES (AIPTS) 2024 – TEST 10 (CURRENT AFFAIRS – SEPT ’23)

Q.13) Consider the following statements Q.15) Consider the following statements
regarding Fiscal Deficit: regarding the ‘State of Food Security and
1. It is the difference between Nutrition in the World (SOFI) 2023’, a
the government's total report released by the United Nations Food
expenditure and its total and Agriculture Organization (FAO):
revenue including borrowings. 1. The global hunger numbers have
2. A high fiscal deficit represents a remained stagnant between 2021
healthy economy while a low fiscal and 2022 but the number of people
deficit represents a debt burden. facing hunger worldwide has
3. It enables the government to decreased.
increase spending on public 2. Child malnutrition is high as 22.3%
services that can of children were stunted, 6.8%
stimulate economic growth. were wasted, and 5.6% were
How many of the above statements are overweight in 2021.
correct? 3. South Asia recorded the highest
a) Only one number of individuals unable to
b) Only two afford a healthy diet.
c) All three 4. India has the highest cost of a
d) None healthy diet among BRICS
nations and its neighbours.
How many of the given statements are
Q.14) Consider the following statements
correct?
regarding Gabon:
a) Only one
1. It is the largest oil producer in Sub-
b) Only two
Saharan Africa.
c) Only three
2. It is a member of the Organization
d) All four
of the Petroleum Exporting
Countries (OPEC).
3. The Loango National Park is one of Q.16) Consider the following statements
the country’s best wildlife-watching regarding the key findings of the Air Quality
spots. Life Index released by the Energy Policy
How many of the above statements are Institute at the University of Chicago
correct? (EPIC):
a) Only one 1. It mentions pollution as the
b) Only two foremost external threat to human
c) All three health around the world.
d) None 2. Bangladesh, India, Nepal, and
Pakistan are the four most polluted
countries in the world.
3. According to this index, Delhi is the
world’s most polluted city.

https://t.me/visioniastestseries2024
https://upscmaterial.online/

IASBABA’S PRELIMS TEST SERIES (AIPTS) 2024 – TEST 10 (CURRENT AFFAIRS – SEPT ’23)

How many of the above statements are academic practices within a credit
correct? bank.
a) Only one How many of the given statements are
b) Only two correct?
c) All three a) Only one
d) None b) Only two
c) Only three
Q.17) Consider the following statements d) All four
regarding Pulikkali:
1. It is one of the folk art forms Q.19) Consider the following statements
of Telangana. regarding the Gujarat Declaration:
2. The main theme of this folk art is 1. It was adopted at the first World
tiger hunting. Health Organization (WHO)
3. It is performed on the fourth day of Traditional Medicine Global
the Onam festival. Summit.
How many of the above statements are 2. It placed importance on global
correct? commitments towards indigenous
a) Only one knowledge, biodiversity, and
b) Only two traditional, complementary, and
c) All three integrative medicine.
d) None Choose the correct code:
a) 1 only
Q.18) Consider the following statements b) 2 only
regarding the National Education Policy c) Both 1 and 2
2020: d) Neither 1 nor 2
1. Its primary objective is to prioritize
learning in a student’s mother Q.20) Consider the following statements
tongue. regarding the Pradhan Mantri Dakshata
2. It aims to create the National Aur Kushalata Sampanna Hitgrahi(PM-
Research Foundation to facilitate DAKSH) Yojana:
research within academic 1. It is a central sector scheme
institutions. launched by the Ministry of Social
3. It establishes the National Justice & Empowerment (MoSJ&E).
Curriculum Framework for School 2. There is no income limit for SCs,
Education to set specific learning OBCs, EBCs, and SafaiKaramcharis.
standards. 3. The age criterion for the scheme is
4. It establishes a National Credit between 18-45 years.
Framework to encompass various

https://t.me/visioniastestseries2024
https://upscmaterial.online/

IASBABA’S PRELIMS TEST SERIES (AIPTS) 2024 – TEST 10 (CURRENT AFFAIRS – SEPT ’23)

4. The stipend under this scheme is 3. It is the world’s only flightless


Rs.1,500/- per month for all target parrot.
groups. 4. It is the world’s heaviest parrot.
How many of the given statements are How many of the given statements are
correct? correct?
a) Only one a) Only one
b) Only two b) Only two
c) Only three c) Only three
d) All four d) All four

Q.21) Consider the following statements Q.23) Consider the following statements
regarding the Education to regarding the different official tracks under
Entrepreneurship Initiative: G20:
1. It is a collaboration between the 1. The Finance Track is headed by the
Ministry of Education, the Ministry finance ministers and central bank
of Skill Development & governors and mainly focuses on
Entrepreneurship, and France. issues of the global economy.
2. It aims to provide digital marketing 2. The Sherpa Track is headed by civil
skills training to five lakh or engagement groups and mainly
entrepreneurs over the next three focuses on socio-economic issues.
years. Choose the correct code:
3. Under this initiative, training will be a) 1 only
conducted in seven regional b) 2 only
languages by using platforms like c) Both 1 and 2
Facebook, WhatsApp, and d) Neither 1 nor 2
Instagram.
How many of the above statements are
Q.24) The Johannesburg Declaration and
correct?
the Contingent Reserve Arrangement are
a) Only one
related to which of the following
b) Only two
organisation?
c) All three
a) Association of Southeast Asian
d) None
Nations
b) World Bank
Q.22) Consider the following statements c) Asian Development Bank
regarding the ‘Kakapo’: d) BRICS
1. It is a species of parrot found only
in Australia.
2. It is listed as critically endangered
on the IUCN List.

https://t.me/visioniastestseries2024
https://upscmaterial.online/

IASBABA’S PRELIMS TEST SERIES (AIPTS) 2024 – TEST 10 (CURRENT AFFAIRS – SEPT ’23)

Q.25) Consider the following statements 3. There are currently no drugs or


regarding oil production in India: vaccines specific to treat this
1. India is the world’s largest infection.
consumer and the world’s largest How many of the above statements are
importer of vegetable oil. correct?
2. In terms of volumes, palm oil, soya a) Only one
bean oil, and mustard oil are b) Only two
the three largest consumed edible c) All three
oils in India. d) None
Choose the correct code:
a) 1 only
Q.28) Consider the following statements
b) 2 only
regarding the Northern Sea Route (NSR):
c) Both 1 and 2
1. It connects the eastern and western
d) Neither 1 nor 2
regions of the Pacific Ocean.
2. It is the shortest shipping route
Q.26) Consider the following statements connecting Europe and the Asia-
regarding Bhoj Wetland: Pacific region.
1. It is a Ramsar site located in Uttar Choose the correct code:
Pradesh. a) 1 only
2. It consists of two contiguous b) 2 only
human-made reservoirs. c) Both 1 and 2
3. It was first conceived by the King d) Neither 1 nor 2
Paramara Raja Bhoj.
How many of the above statements are
Q.29) Consider the following pairs:
correct?
a) Only one
Exquisite product State
b) Only two
1. Longpi Pottery Gujarat
c) All three
d) None 2. Sheep Wool Himachal
Stoles Pradesh
Q.27) Consider the following statements 3. Meenakari Rajasthan
regarding the Nipah Virus (NiV):
4. Araku Valley Andhra
1. It is a zoonotic virus with its natural
Coffee Pradesh
host being fruit bats.
2. It is listed as a priority disease for How many pairs are correctly matched?
the WHO Research and a) One pair
Development Blueprint. b) Two pairs
c) Three pairs
d) Four pairs

https://t.me/visioniastestseries2024
https://upscmaterial.online/

IASBABA’S PRELIMS TEST SERIES (AIPTS) 2024 – TEST 10 (CURRENT AFFAIRS – SEPT ’23)

Q.30) It is the only African country with Q.32) Consider the following statements
coastal exposure to both the Atlantic regarding Recorded Forest Area (RFA):
Ocean and the Mediterranean Sea. It lies 1. It refers to lands that are recorded
directly across the Strait of Gibraltar from as forests, regardless of their official
Spain. Its national language is Arabic. It has classification or ownership.
a Mediterranean climate, with mild wet 2. It does not include notified forests
winters and hot dry summers. It has and unclassed forests.
Algeria to the east and southeast, Western 3. Over half of the Northeast region is
Sahara to the south, the Atlantic Ocean to categorized as RFA.
the west, and the Mediterranean Sea to How many of the above statements are
the north. correct?
The above paragraph describes which of a) Only one
the following country? b) Only two
a) Tunisia c) All three
b) Nigeria d) None
c) Ghana
d) Morocco
Q.33) Consider the following statements:
Statement I:
Q.31) Consider the following statements
Several banks have enabled the
regarding the Central Bank Digital Currency
interoperability of the Unified Payments
(CBDC):
Interface’s (UPI) Quick Response (QR) code
1. It is fiat money backed by a
with their Central Bank Digital Currency
government through its central
(CBDC).
bank in a virtual form.
2. It combines the power of Statement II:
blockchain with the logistics of It ensures seamless transactions between a
distributed ledger technology (DLT). customer and merchant without needing
3. Just like paper currency, each unit to switch between multiple digital
is uniquely identifiable to prevent platforms.
counterfeiting.
Which one of the following is correct with
How many of the above statements are respect to the above statements?
correct? a) Both Statement I and Statement II
a) Only one are correct and Statement II is the
b) Only two correct explanation for Statement I
c) All three b) Both Statement I and Statement II
d) None are correct and Statement II is not
the correct explanation' for
Statement I
c) Statement I is correct but
Statement II is incorrect

https://t.me/visioniastestseries2024
https://upscmaterial.online/

IASBABA’S PRELIMS TEST SERIES (AIPTS) 2024 – TEST 10 (CURRENT AFFAIRS – SEPT ’23)

d) Statement I is incorrect Statement II a) 1 only


is correct b) 2 only
c) Both 1 and 2
d) Neither 1 nor 2
Q.34) Consider the following statements
regarding the “Assessment Report on
Invasive Alien Species and their Control’’ Q.36) Which of the following countries
released by the Intergovernmental participate in the East Asia Summit?
Platform on Biodiversity and Ecosystem 1. Australia
Services (IPBES): 2. Indonesia
1. The water hyacinth is the world’s 3. Cambodia
most widespread invasive alien 4. United States of America
species on land. Choose the correct code:
2. Invasive alien species contribute to a) Only one
60% of global plant and animal b) Only two
extinctions. c) Only three
3. The annual costs of invasive alien d) All four
species have quadrupled every
decade since 1970.
Q.37) Consider the following statements
How many of the above statements are
regarding the Malaviya Mission:
correct?
1. It is a teachers' training programme
a) Only one
by the University Grants
b) Only two
Commission.
c) All three
2. It aims to improve the capacity
d) None
building of faculty members in
primary schools.
Q.35) Consider the following statements: Choose the correct code:
1. Battery storage or battery energy a) 1 only
storage systems (BESS) are devices b) 2 only
that enable energy from c) Both 1 and 2
renewables to be stored and then d) Neither 1 nor 2
released when the power is needed
most.
2. The Scheme for Viability Gap Q.38) Consider the following statements:
Funding (VGF) for the development 1. The Blue Button Movement is
of Battery Energy Storage Systems organised by the World Health
(BESS) envisages the development Organisation on World Cancer Day.
of 4,000 MWh of BESS projects by 2. DemClinic will be the first national
2049-50. comprehensive expert-led
cognitive assessment platform for
Choose the correct code:
the elderly.

https://t.me/visioniastestseries2024
https://upscmaterial.online/

IASBABA’S PRELIMS TEST SERIES (AIPTS) 2024 – TEST 10 (CURRENT AFFAIRS – SEPT ’23)

Choose the correct code: b) Prison Reforms


a) 1 only c) Goods and Service tax
b) 2 only d) None
c) Both 1 and 2
d) Neither 1 nor 2
Q.43) Which of the following are the
objectives of PM Gati Shakti?
Q.39) The Global Biofuel Alliance (GBA) is 1. Establish synergies among different
backed by which of the following ministries during the planning stage
countries? itself
a) Russia, China, and Japan 2. Align timelines of various projects
b) Brazil, Japan, and the USA through efficient planning
c) Brazil, India, and the USA 3. Promote standardization of the
d) Japan, China, and Australia designs for efficient project
approval
4. Enable standardized procedures by
Q.40) HII-A rocket to explore the origins of
all the agencies to reduce project
the universe is launched by:
delays
a) Germany
b) China How many of the given statements are
c) India correct?
d) Japan a) Only one
b) Only two
c) Only three
Q.41) Consider the following statements d) All four
regarding the Konark Sun Temple:
1. It was constructed during the reign
of King Narasimhadeva I of Q.44) Recently, a white sambar was
the Eastern Ganga Dynasty. photographed in the Cauvery Wildlife
2. It was built using Khondalite rocks Sanctuary which was constituted in 1989
and was declared a UNESCO World under the Wildlife Protection Act of 1972.
Heritage Site in 1984. It is located in which state of India?
a) Tamil Nadu
Choose the correct code:
b) Kerala
a) 1 only
c) Karnataka
b) 2 only
d) Goa
c) Both 1 and 2
d) Neither 1 nor 2
Q.45) Basket Sneaking, False Urgency,
Subscription Trap, and Nagging are the
Q.42) Amitava Roy Committee formed by
terms related to which of the following?
the Supreme Court of India is related to:
a) Cryptocurrency
a) Other Backward Classes
b) World Bank

https://t.me/visioniastestseries2024
https://upscmaterial.online/

IASBABA’S PRELIMS TEST SERIES (AIPTS) 2024 – TEST 10 (CURRENT AFFAIRS – SEPT ’23)

c) Moon Q.48) Consider the following statements


d) Dark patterns regarding ‘Bharat: The Mother of
Democracy’ Portal:
1. It was launched by the Ministry of
Q.46) Consider the following statements
Culture on the eve of the G20
regarding the Incremental cash reserve
leadership summit.
ratio (I-CRR):
2. It offers content that delves into
1. It is an additional cash balance that
India’s democratic legacy spanning
banks can ask the Reserve Bank of
an astonishing 7,000 years.
India to maintain over and above
3. The content on the portal is
the Cash Reserve Ratio(CRR).
available in 16 languages including
2. It was announced as a temporary
German, French, Spanish, and
measure to absorb excess liquidity
Arabic.
from the banking system.
How many of the above statements are
Choose the correct code:
correct?
a) 1 only
a) Only one
b) 2 only
b) Only two
c) Both 1 and 2
c) All three
d) Neither 1 nor 2
d) None

Q.47) Consider the following statements


Q.49) Consider the following statements:
regarding Hallmarking:
1. Himachal Pradesh became the first
1. It is the accurate determination and
State in the country to legalize
official recording of the
cannabis cultivation.
proportionate content of precious
2. Hemp is a botanical class
metals in precious metal articles.
of cannabis sativa cultivars grown
2. At present, only two precious
specifically for industrial or
metals – gold and silver come under
medicinal use.
hallmarking in India.
Choose the correct code:
Choose the correct code:
a) 1 only
a) 1 only
b) 2 only
b) 2 only
c) Both 1 and 2
c) Both 1 and 2
d) Neither 1 nor 2
d) Neither 1 nor 2

Q.50) Consider the following statements:


Statement I:
The hybrid nanoparticles made of gold and
copper sulphide can kill cancer cells.

https://t.me/visioniastestseries2024
https://upscmaterial.online/

IASBABA’S PRELIMS TEST SERIES (AIPTS) 2024 – TEST 10 (CURRENT AFFAIRS – SEPT ’23)

Statement II: Q.52) Which of the following are


They can create heat and release toxic probiotic-rich foods?
oxygen when illuminated with light, 1. Buttermilk
effectively targeting and killing cancer cells. 2. Cottage cheese
3. Kombucha
Which one of the following is correct with
4. Tempeh
respect to the above statements?
a) Both Statement I and Statement II How many of the above option/s are
are correct and Statement II is the correct?
correct explanation for Statement I a) Only one
b) Both Statement I and Statement II b) Only two
are correct and Statement II is not c) Only three
the correct explanation' for d) All four
Statement I
c) Statement I is correct but Q.53) Consider the following statements:
Statement II is incorrect
Statement I:
d) Statement I is incorrect Statement II
is correct The Unified Portal for Agricultural
Statistics (UPAg) is an advanced
agricultural data management platform
Q.51) Consider the following statements
that provides standardized agricultural
about the National eVidhan Application
data and agricultural data analysis.
(NeVA)
1. Its objective is to make all Statement II:
legislatures digital with the help of Because it provides real-time information
Information & Communication on crop production, market trends,
Technologies (ICT). pricing, and other vital agricultural data
2. The Ministry of Electronics and along with giving information on
Information Technology (MietY) production trends, trade correlations, and
provides funding and technical consumption patterns, aiding
assistance to the project. policymakers in making informed
3. Himachal Pradesh was the first decisions.
Digital Legislature in the country. Which one of the following is correct with
How many of the above statements are respect to the above statements?
correct? a) Both Statement I and Statement II
a) Only one are correct and Statement II is the
b) Only two correct explanation for Statement I
c) All three b) Both Statement I and Statement II
d) None are correct and Statement II is not
the correct explanation' for
Statement I

https://t.me/visioniastestseries2024
https://upscmaterial.online/

IASBABA’S PRELIMS TEST SERIES (AIPTS) 2024 – TEST 10 (CURRENT AFFAIRS – SEPT ’23)

c) Statement I is correct but 3. India has phased out


Statement II is incorrect chlorofluorocarbons, carbon
d) Statement I is incorrect Statement tetrachloride, halons,
II is correct hydrofluorocarbons, methyl
bromide, and methyl chloroform
following the Montreal Protocol.
Q.54) Which of the following are the
objectives of the Pradhan Mantri Matasya How many of the above statements are
Sampada Yojna (PMMSY)? correct?
1. It harnesses fisheries'' potential in a) Only one
a sustainable, responsible, b) Only two
inclusive, and equitable manner c) All three
and provides for social, physical, d) None
and economic security for fishers
and fish farmers. Q.56) Consider the following statements
2. It enhances fish production and about Santiniketan
productivity through expansion, 1. It was originally an ashram built by
intensification, diversification, and Debendranath Tagore where
productive utilization of land and anyone, irrespective of caste and
water. creed, could come and spend time
3. It modernizes and strengthens the meditating on the one Supreme
value chain, post-harvest God.
management, and quality 2. It represents the British colonial
improvement. architectural orientations of the
How many of the above statements are early 20th century and European
correct? modernism.
a) Only one 3. It is adorned with splendid
b) Only two sculptures, frescoes, murals, and
c) All three paintings created by Rabindranath
d) None Tagore, Nandalal Bose, and
Ramkinkar Baij.
4. It has been added to the World
Q.55) Consider the following statements:
Heritage Site List recognized by the
1. World Ozone Day is celebrated on
United Nations Educational,
the 16th of September each year
Scientific and Cultural Organization
to commemorate the signing of
(UNESCO).
the Kyoto Protocol.
2. The Kigali Agreement aims How many of the given statements are
to phase down hydrofluorocarbons correct?
(HFCs) by curbing both their a) Only one
production and consumption. b) Only two
c) Only three

https://t.me/visioniastestseries2024
https://upscmaterial.online/

IASBABA’S PRELIMS TEST SERIES (AIPTS) 2024 – TEST 10 (CURRENT AFFAIRS – SEPT ’23)

d) All four Q.59) Consider the following statements:


1. India is a member of the
International Organization of Legal
Q.57) Consider the following statements
Metrology (OIML), an international
about Scrub typhus
standard-setting body that
1. It is a disease caused by a fungus
develops model regulations and
called Orientia tsutsugamushi.
standards.
2. It spreads to people through bites
2. World Metrology Day is celebrated
of infected chiggers.
in commemoration of the
3. It will spread from person to
anniversary of the signing of
person and is curable with a
the Metre Convention in 1875.
vaccine.
3. The theme for World
4. It should be treated with the
Meteorological Day 2023 is “The
antibiotic doxycycline.
Future of Weather, Climate, and
How many of the given statements are Water Across Generations”.
correct?
How many of the above statements are
a) Only one
correct?
b) Only two
a) Only one
c) Only three
b) Only two
d) All four
c) All three
d) None
Q.58) Consider the following statements:
1. The African cheetah is listed as
Q.60) Consider the following statements
vulnerable on the IUCN List.
about the Business Responsibility and
2. The Asiatic cheetah is listed as
Sustainability Reporting (BRSR)
critically endangered on the IUCN
1. It is a voluntary disclosure
List.
mechanism for the top 1000 listed
3. The Asiatic cheetah is smaller and
companies or businesses to report
paler than the African cheetah.
their performance
4. Both the Asiatic cheetah and
on environmental, social, and
African cheetah are listed in
corporate governance (ESG)
Appendix I of CITES.
parameters.
How many of the given statements are 2. It is rooted in the nine principles
correct? outlined in the National Guidelines
a) Only one for Responsible Business Conduct
b) Only two (NGRBC).
c) Only three 3. The Indian Institute of Corporate
d) All four Affairs (IICA) is registered as a
society under the Societies
Registration Act of 1860 and works

https://t.me/visioniastestseries2024
https://upscmaterial.online/

IASBABA’S PRELIMS TEST SERIES (AIPTS) 2024 – TEST 10 (CURRENT AFFAIRS – SEPT ’23)

under the aegis of the Ministry of c) All three


Corporate Affairs. d) None
How many of the above statements are
correct? Q.63) Consider the following statements:
a) Only one
Statement I:
b) Only two
c) All three Deoxygenation in water bodies results in
d) None dead zones leading to mass fish kills and
other marine organism die-offs.
Statement II:
Q.61) Consider the following statements:
1. Skills Acquisition and Knowledge Because deoxygenation leads to the
Awareness for Livelihood reduction or depletion of dissolved oxygen
(SANKALP) is a central sector levels in aquatic environments.
scheme that aims to create Which one of the following is correct with
convergence among all skill respect to the above statements?
training activities. a) Both Statement I and Statement II
2. Skills Strengthening for Industrial are correct and Statement II is the
Value Enhancement (STRIVE) is a correct explanation for Statement I
centrally sponsored scheme that b) Both Statement I and Statement II
aims to improve the relevance and are correct and Statement II is not
efficiency of skills training provided the correct explanation' for
through Industrial Training Statement I
Institutes (ITIs). c) Statement I is correct but
Choose the correct code: Statement II is incorrect
a) 1 only d) Statement I is incorrect Statement
b) 2 only II is correct
c) Both 1 and 2
d) Neither 1 nor 2
Q.64) Consider the following statements
about the Export Promotion Council for
Q.62) Which of the following initiatives are Handicrafts (EPCH)
related to marine pollution? 1. It is a non-profit organisation that
1. UN Convention on the Law of the aims to promote, support, protect,
Sea maintain, and increase the export
2. London Convention of handicrafts.
3. International Convention for the 2. It is an apex body of handicrafts
Prevention of Pollution from Ships exporters for the promotion of
exports of handicrafts from the
Choose the correct code:
country.
a) Only one
b) Only two Choose the correct code:

https://t.me/visioniastestseries2024
https://upscmaterial.online/

IASBABA’S PRELIMS TEST SERIES (AIPTS) 2024 – TEST 10 (CURRENT AFFAIRS – SEPT ’23)

a) 1 only How many of the given statements are


b) 2 only correct?
c) Both 1 and 2 a) Only one
d) Neither 1 nor 2 b) Only two
c) Only three
d) All four
Q.65) Consider the following statements
about Sickle Cell Disease (SCD)
1. It is a group of inherited white Q.67) The planetary boundaries
blood cell disorders with framework was first proposed by Johan
symptoms such as chronic anaemia Rockstrom to define the environmental
and delayed puberty. limits within which humanity can safely
2. It can be treated through blood operate to maintain Earth's stability and
transfusions, stem cell biodiversity. Which of the following are
transplantation, and hydroxyurea. included in the nine planetary
3. The National Sickle Cell Anemia boundaries?
Eradication Mission aims to 1. Ocean acidification
eliminate sickle cell anemia in India 2. Biogeochemical flows
by 2050. 3. Freshwater use
How many of the above statements are 4. Climate change
correct? How many of the given statements are
a) Only one correct?
b) Only two a) Only one
c) All three b) Only two
d) None c) Only three
d) All four

Q.66) Consider the following statements:


1. Rubber requires deep and lateritic Q.68) Consider the following statements
fertile soil with an acidic pH of 4.5 regarding the 7th report on antimicrobial
to 6.0. use in animals released by the World
2. India is currently the world’s third- Organisation for Animal Health (WOAH)
largest producer of natural rubber. 1. There is a 30% increase in global
3. Rubber is used in the antimicrobial usage in
manufacturing of tyres, medical animals from 2017 to 2019.
devices, sporting goods, and 2. The common antimicrobial growth
footwear. promoters are flavomycin,
4. The Rubber Board is a statutory bacitracin, avilamycin, and tylosin.
organization constituted under the Choose the correct code:
Rubber Act of 1947. a) 1 only
b) 2 only

https://t.me/visioniastestseries2024
https://upscmaterial.online/

IASBABA’S PRELIMS TEST SERIES (AIPTS) 2024 – TEST 10 (CURRENT AFFAIRS – SEPT ’23)

c) Both 1 and 2 Q.71) Consider the following statements:


d) Neither 1 nor 2 1. Chennakeshava Temple is a 12th-
century Hindu temple dedicated to
Vishnu located on the banks of the
Q.69) Consider the following statements:
Yagachi River.
1. According to a recent study by the
2. Hoysaleswara temple is a 12th-
Journal Nature Communications,
century Hindu temple dedicated to
lion habitats have seen a 60%
Shiva located on the banks of the
decline in West and Central Africa.
Dwarasamudra tank.
2. Asiatic lions are slightly smaller
than African lions and are listed as Choose the correct code:
vulnerable on the IUCN List. a) 1 only
b) 2 only
Choose the correct code:
c) Both 1 and 2
a) 1 only
d) Neither 1 nor 2
b) 2 only
c) Both 1 and 2
d) Neither 1 nor 2 Q.72) Consider the following statements
regarding the relevance of the Non-
Alignment Movement
Q.70) It is a west-flowing river of the
1. It acted as a protector for small
peninsular region flowing through a rift
countries against Western
valley between the Vindhya Range on the
hegemony.
north and the Satpura Range on the south.
2. It has played an active role in
Its right tributaries are Hiran, Tendori,
preserving world peace.
Barna, Kolar, Man, Uri, Hatni, and Orsang
3. It acts as a bridge between the
while its left tributaries are Burner, Banjar,
political and ideological differences
Sher, Shakkar, Dudhi, Tawa, Ganjal, Chhota
existing in the international
Tawa, Kundi, Goi, and Karjan. It serves as a
environment.
traditional boundary between North and
South India. Its major dams include How many of the above statements are
Omkareshwar and Maheshwar. It correct?
originates from the Amarkantak peak of a) Only one
Maikal Mountain and flows into the Gulf b) Only two
of Khambhat. c) All three
d) None
The above paragraph refers to which of
the following river?
a) Krishna Q.73) Consider the following statements:
b) Godavari Statement I:
c) Tapi
Food fortification began in India in the
d) Narmada
1950s and it helps to combat hidden
hunger.

https://t.me/visioniastestseries2024
https://upscmaterial.online/

IASBABA’S PRELIMS TEST SERIES (AIPTS) 2024 – TEST 10 (CURRENT AFFAIRS – SEPT ’23)

Statement II: 3. Canker Tomato


Because it adds essential vitamins and
minerals to increase the nutritional value 4. Downy Grapes
of common foods. Mildew

Which one of the following is correct with How many given pairs are correctly
respect to the above statements? matched?
a) Both Statement I and Statement II a) One pair
are correct and Statement II is the b) Two pairs
correct explanation for Statement I c) Three pairs
b) Both Statement I and Statement II d) Four pairs
are correct and Statement II is not
the correct explanation' for Q.76) Consider the following statements
Statement I regarding the Times Higher
c) Statement I is correct but Education’s World University Rankings
Statement II is incorrect 2024
d) Statement I is incorrect Statement 1. The US is the most-represented
II is correct country while India is the fourth
most-represented country in terms
Q.74) Consider the following statements of institutions.
about Battery energy storage systems 2. Jamia Millia Islamia is the top
(BESS) university in India while the
1. It preserves energy generated from University of Oxford tops the
non-renewable sources. overall ranking list.
2. It overcomes the issue of Choose the correct code:
intermittent generation patterns of a) 1 only
renewable energy sources. b) 2 only
Choose the correct code: c) Both 1 and 2
a) 1 only d) Neither 1 nor 2
b) 2 only
c) Both 1 and 2 Q.77) CALIPSO Mission was launched in
d) Neither 1 nor 2 2006 to enhance our understanding of
clouds and aerosols and their roles in
Q.75) Consider the following pairs: Earth’s climate system. It was jointly
launched by which of the following
Disease Plant
countries?
1. Red Rot a) India and the U.S.
Sugarcane
b) France and Germany
2. Pink c) France and the U.S.
Cotton
Bollworm d) India and Japan

https://t.me/visioniastestseries2024
https://upscmaterial.online/

IASBABA’S PRELIMS TEST SERIES (AIPTS) 2024 – TEST 10 (CURRENT AFFAIRS – SEPT ’23)

Q.78) Consider the following statements Q.80) Consider the following statements
regarding the Battle of Haifa regarding the Central Board of Indirect
1. It was fought during the Second Taxes and Customs (CBIC)
World War and is commemorated 1. It is a part of the Department of
as Hafia Day on 23rd September, Revenue under the Ministry of
since 2010. Finance.
2. It involved the British Empire 2. It is a statutory body established
fighting against the Ottoman under the Central Boards of
Kingdom, the German Empire, and Revenue Act, of 1963.
Austria-Hungary. 3. It is the nodal national
Choose the correct code: agency responsible for
a) 1 only administering customs, central
b) 2 only excise, and narcotics in India.
c) Both 1 and 2 How many of the above statements are
d) Neither 1 nor 2 correct?
a) Only one
b) Only two
Q.79) Consider the following statements
c) All three
regarding the Dadasaheb Phalke Lifetime
d) None
Achievement Award
1. It is India’s highest cinema honour
presented at the National Film Q.81) Consider the following statements
Awards ceremony. regarding the Global Innovation Index
2. Devika Rani is the first awardee of 2023
this award which was initiated in 1. It is published annually by the
1969. World Intellectual Property
3. Its prize includes a Swarna Kamal Organization.
(Golden Lotus) medallion, a shawl, 2. Switzerland is the most innovative
and a cash prize of 25 lakh. economy followed by Sweden,
How many of the above statements are the United States, and the United
correct? Kingdom.
a) Only one 3. Shenzhen–Hong Kong–Guangzhou
b) Only two is the top science and technology
c) All three innovation cluster in the world.
d) None How many of the above statements are
correct?
a) Only one
b) Only two
c) All three
d) None

https://t.me/visioniastestseries2024
https://upscmaterial.online/

IASBABA’S PRELIMS TEST SERIES (AIPTS) 2024 – TEST 10 (CURRENT AFFAIRS – SEPT ’23)

Q.82) Consider the following statements b) Both Statement I and Statement II


regarding the Psyche mission are correct and Statement II is not
1. It is an ISRO mission to study a the correct explanation' for
metal-rich asteroid named Psyche. Statement I
2. It is powered by solar electric c) Statement I is correct but
propulsion and has Statement II is incorrect
a magnetometer, a gamma-ray, d) Statement I is incorrect Statement
and a neutron spectrometer. II is correct
3. The mission helps to understand
the early solar system and
Q.84) Consider the following statements:
the formation of terrestrial
1. Valmiki TIGER RESERVE IS THE
planets.
ONLY TIGER RESERVE IN BIHAR.
4. Asteroid Psyche is located in
2. Karnataka has the highest number
the main asteroid belt between
of tiger reserves.
Mars and Jupiter.
3. Kamlang Tiger Reserve is the
How many of the given statements are largest tiger reserve in India.
correct? 4. Bhor Tiger Reserve is the smallest
a) Only one tiger reserve in India.
b) Only two
How many of the given statements are
c) Only three
correct?
d) All four
a) Only one
b) Only two
Q.83) Consider the following statements: c) Only three
Statement I: d) All four

Cabomba furcata is economically and


ecologically hindering the growth of native Q.85) Consider the following statements
aquatic plants and freshwater fish in about Nilgiri tahr
Kerala. 1. It is the state animal of Tamil Nadu.
Statement II: 2. It is the only mountain ungulate in
southern India.
Its active stem propagation prevents light
3. It is endemic to the southern part
from penetrating the surface of water and
of the Western Ghats.
it requires a huge quantity of oxygen,
4. It is protected under Schedule I of
resulting in a decline in biodiversity.
the Wildlife Protection Act of 1972.
Which one of the following is correct with
How many of the given statements are
respect to the above statements?
correct?
a) Both Statement I and Statement II
a) Only one
are correct and Statement II is the
b) Only two
correct explanation for Statement I
c) Only three

https://t.me/visioniastestseries2024
https://upscmaterial.online/

IASBABA’S PRELIMS TEST SERIES (AIPTS) 2024 – TEST 10 (CURRENT AFFAIRS – SEPT ’23)

d) All four banking, and third-party fund


transfers.
2. It offers 3 types of saving accounts
Q.86) Consider the following:
- Regular Account – Safal, Basic
Savings Bank Deposit Account
Form of Painting Work (BSBDA) – Sugam, and BSBDA
1. Malwa Small – Saral.
Rasikapriya
Painting 3. The maximum limit on deposits for
2. Mewar current and savings accounts is Rs
Bhairavi Ragini
Painting 5 lakh and they cannot issue debit
3. Bundi cards and ATM cards.
Ragamala
Painting How many of the above statements are
How many pairs are correctly matched? correct?
a) One pair a) Only one
b) Two pairs b) Only two
c) Three pairs c) All three
d) None d) None

Q.87) Consider the following statements: Q.89) Consider the following statements
1. Gurez Valley is known as the valley regarding the Unified Registration Portal
of wild tulips. for GOBARdhan
2. Burzil Pass connects the Kashmir 1. It is launched by the Ministry of
valley with the Deosai plains of Rural Development.
Ladakh. 2. It aims to streamline the
3. Kaobal Gali connects Gurez Valley registration of compressed bio-gas
to Mushkoh Valley. and biogas plants nationwide.
3. It provides real-time tracking,
How many of the above statements are
ensuring transparency, and social
correct?
accountability of the scheme.
a) Only one
b) Only two How many of the above statements are
c) All three correct?
d) None a) Only one
b) Only two
c) All three
Q.88) Consider the following statements d) None
about the India Post Payments Bank
Limited (IPPB)
1. It accepts deposits and offers
remittance services, mobile

https://t.me/visioniastestseries2024
https://upscmaterial.online/

IASBABA’S PRELIMS TEST SERIES (AIPTS) 2024 – TEST 10 (CURRENT AFFAIRS – SEPT ’23)

Q.90) Consider the following pairs: c) Statement I is correct but


Statement II is incorrect
d) Statement I is incorrect Statement
Rhino Species IUCN Status
II is correct
1. Javan Rhinos Critically
Endangered Q.92) Consider the following statements:
2. Sumatran Critically 1. Kallanai Dam in Kerala is the oldest
Rhinos Endangered dam in India.
3. Greater One- Critically 2. Tehri Dam in Uttarakhand is the
Horned Endangered tallest dam in India.
Rhinos 3. Hirakud Dam in Jharkhand is the
4. White Rhinos longest dam in India.
Near
Threatened How many of the above statements are
correct?
How many pairs are correctly matched?
a) Only one
a) One pair
b) Only two
b) Two pairs
c) All three
c) Three pairs
d) None
d) None

Q.93) Consider the following statements:


Q.91) Consider the following statements:
1. Hormones produced by the
Statement I: adrenal cortex include cortisol and
India will be included in the Government androgens.
Bond Index-Emerging Markets (GBI-EM) 2. Hormones produced by the thyroid
published by JP Morgan Chase & Co from gland include calcitonin and
June 2024. triiodothyronine.
Statement II: 3. Hormones produced by the
endocrine pancreas include
It acts as a pull factor to prompt foreign
somatostatin and insulin.
inflows into India and it increases the cost
4. Hormones produced by the gonads
of borrowing for the government.
include Testosterone and estrogen.
Which one of the following is correct with
How many of the given statements are
respect to the above statements?
correct?
a) Both Statement I and Statement II
a) Only one
are correct and Statement II is the
b) Only two
correct explanation for Statement I
c) Only three
b) Both Statement I and Statement II
d) All four
are correct and Statement II is not
the correct explanation' for
Statement I

https://t.me/visioniastestseries2024
https://upscmaterial.online/

IASBABA’S PRELIMS TEST SERIES (AIPTS) 2024 – TEST 10 (CURRENT AFFAIRS – SEPT ’23)

Q.94) All India Financial Institutions (AIFIs) Q.96) Which of the following countries are
is a group composed of financial part of Five Eyes Alliance?
regulatory bodies that play a pivotal role 1. India
in the financial markets by assisting the 2. Australia
proper allocation of resources, sourcing 3. New Zealand
from businesses that have a surplus and 4. United Kingdom
distributing to others who have deficits. Choose the correct code:
Which of the following AIFIs in India are a) Only one
regulated by the Reserve Bank of India? b) Only two
1. National Bank for Financing c) Only three
Infrastructure and Development d) All four
(NaBFID)
2. National Housing Bank (NHB)
3. Small Industries Development Q.97) Consider the following statements:
Bank of India (SIDBI) 1. An artificial reef is a manmade
4. Export-Import Bank of India (EXIM structure that may mimic some of
Bank) the characteristics of a natural reef.
2. Submerged shipwrecks, oil and gas
How many of the given statements are
platforms, bridges, and lighthouses
correct?
function as artificial reefs.
a) Only one
b) Only two Choose the correct code:
c) Only three a) 1 only
d) All four b) 2 only
c) Both 1 and 2
d) Neither 1 nor 2
Q.95) Consider the following statements
about the Indian Skimmer
1. It is listed as critically endangered Q.98) Consider the following statements:
on the IUCN List. 1. The World Federation of the Deaf
2. It breeds between February to (WFD) is an international non-
June and raise one to three chicks profit and non-governmental
per clutch. organization of deaf associations.
3. It is protected under Schedule I of 2. The World Federation of the Deaf
the Wildlife Protection Act of 1972. (WFD) has proclaimed 23rd
September as the International
How many of the above statements are
Day of Sign Languages.
correct?
3. The Theme for the International
a) Only one
Day of Sign Languages 2023 is “A
b) Only two
World where Deaf People
c) All three
Everywhere can Sign Anywhere!”.
d) None

https://t.me/visioniastestseries2024
https://upscmaterial.online/

IASBABA’S PRELIMS TEST SERIES (AIPTS) 2024 – TEST 10 (CURRENT AFFAIRS – SEPT ’23)

How many of the above statements are 2. It is a statutory body under the
correct? National Medical Commission Act,
a) Only one 2019.
b) Only two 3. Under-Graduate Medical Education
c) All three Board is one among the five
d) None autonomous boards under NMC.
How many of the above statements are
Q.99) Consider the following statements: correct?
1. Greenwishing refers to a) Only one
organisations expressing a desire b) Only two
to be more environmentally c) All three
responsible without taking d) None
concrete actions to achieve those
goals.
2. Greenwashing refers to the
deceptive practice of making false
or exaggerated claims about the
environmental friendliness of a
company’s products, services, or
practices.
3. Greenhushing implies a situation
where an organisation
intentionally downplays their
positive environmental
achievements.
How many of the above statements are
correct?
a) Only one
b) Only two
c) All three
d) None

Q.100) Consider the following statements


about the National Medical Commission
(NMC)
1. It has replaced the Medical Council
of India (MCI) constituted under
the Indian Medical Council Act,
1956.

https://t.me/visioniastestseries2024
https://upscmaterial.online/

IASBABA’S PRELIMS TEST SERIES (AIPTS) 2024 – TEST 10 (CURRENT AFFAIRS – SEPT ’23)

Q.1) Solution (b)

Statement Analysis:

• Red Sand Boa is an ovoviviparous non-venomous snake. Hence statement 1 is


incorrect.
• Ovoviviparous means they produce young ones by means of eggs which are hatched
within the body of the parent.
• It is predominantly nocturnal.
• Nocturnal means most active during the night.
• It is found in arid and semi-arid regions. Hence statement 2 is correct.
• It is found in India, Pakistan, Sri Lanka, and parts of Bangladesh.
• It has applications in medicine and cosmetics. Hence statement 3 is correct.
• It plays a significant role in the ecosystem by maintaining a healthy population
between prey and predator.
• It is listed as near threatened in the IUCN List. Hence statement 4 is incorrect.
• It is protected under the Appendix II of CITES and Schedule IV of the Wildlife Protection
Act of 1972.

Source: https://www.thehindu.com/sci-tech/energy-and-environment/172-incidents-of-
seizures-of-red-sand-boa-recorded-from-2016-to-2021-wcs-india-
report/article67249299.ece

Q.2) Solution (a)

Statement Analysis:

• Africa holds the potential to emerge as a prominent force in renewable energy


generation. Hence statement 1 is correct.
• It is endowed with a rich array of renewable energy sources, including substantial
solar capacity, wind resources, geothermal areas, hydro energy, and green
hydrogen prospects.
• It also possesses over 40% of the world's crucial mineral reserves crucial for
renewable and low-carbon technologies.
• According to the reports by IRENA, it boasts a solar PV technical potential of 7,900 GW,
hydropower potential of 1,753 GW, and wind energy potential of 461 GW along with
geothermal energy potential. Hence statement 2 is correct.
Source:
https://pib.gov.in/PressReleaseIframePage.aspx?PRID=1953770#:~:text=Singh%2C%20who%
20joined%20the%20meeting,Comoros%20and%20three%20in%20Mali

https://t.me/visioniastestseries2024
https://upscmaterial.online/

IASBABA’S PRELIMS TEST SERIES (AIPTS) 2024 – TEST 10 (CURRENT AFFAIRS – SEPT ’23)

Q.3) Solution (b)

Statement Analysis:

• Bharat Stage (BS) norms are emission standards instituted by the Government of India
to regulate the output of air pollutants from motor vehicles. Hence statement 1 is
correct.
• They are based on the European emission standards and are implemented by
the Central Pollution Control Board in India.
• The fuel standard in the country is decided by the Ministry of Environment, Forest,
and Climate Change. Hence statement 2 is correct.
• Every newly sold and registered vehicle in India is required to adhere to the BS-VI
version of emission regulations. Hence statement 3 is incorrect.
• In BS-VI fuel, the volume of PM 2.5 ranges from 20 to 40 micrograms per cubic metre
whereas in BS-IV fuel it is up to 120 micrograms per cubic metre.
• BS-VI fuel will bring down sulphur content by 5 times from the current BS-IV levels. It
has 10 ppm of sulphur as against 50 ppm in BS-IV.

Note:
• Flex-fuel vehicles (FFV) have engines that can run on flexible fuel - a combination of
Petrol/Diesel/Electric and ethanol, which can include up to 100% ethanol.
• Electrified Flex Fuel Vehicle integrates both a Flex Fuel engine and an electric
powertrain, offering the dual benefit of higher ethanol use and improved fuel
efficiency.
Source: https://pib.gov.in/PressReleasePage.aspx?PRID=1953249

Q.4) Solution (b)

Statement Analysis:

• There are mainly two schools in Hindu law concerning the law of inheritance -
the Mitakshara and the Dayabhaga.
• In the Mitakshara school of inheritance, the property is inherited by the successors
only if they were born in the family of the property owners. Hence statement 1 is
correct.
• In the Dayabhaga school of inheritance, the property goes to the successors only when
the property owner is dead. Hence statement 2 is correct.
• The Mitakshara school of inheritance is practiced in all of India except in West Bengal
and Assam. Hence statement 3 is incorrect.
• The Dayabhaga school of inheritance is mainly practiced in Assam and West Bengal.
Hence statement 4 is incorrect.

https://t.me/visioniastestseries2024
https://upscmaterial.online/

IASBABA’S PRELIMS TEST SERIES (AIPTS) 2024 – TEST 10 (CURRENT AFFAIRS – SEPT ’23)

Source: https://www.thehindu.com/news/national/children-from-void-voidable-marriages-
are-legitimate-can-claim-rights-in-parents-properties-sc/article67259229.ece

Q.5) Solution (c)

Statement Analysis:

• Under the National Pension System(NPS), a unique Permanent Retirement Account


Number (PRAN) is generated and maintained by the Central Recordkeeping Agency
(CRA) for individual subscribers. Hence statement 1 is correct.
• It is being administered and regulated by the Pension Fund Regulatory and
Development Authority (PFRDA) set up under the PFRDA Act, 2013. Hence statement
2 is correct.
• Under this system, a Tier-I account is a pension account having restricted withdrawals
while Tier-II is a voluntary account that offers liquidity of investments and withdrawals.
Hence statement 3 is correct.
• Tier-II account is allowed only when there is an active Tier-I account in the name of the
subscriber. The contributions accumulate over a period of time till retirement grows
with market-linked returns.

Source: https://www.pressreader.com/india/the-hindu-tiruchirapalli-
9WWf/20230902/281891597847368

Q.6) Solution (a)

Statement Analysis:

• The deemed-to-be university is declared by the central government on the advice of


the University Grants Commission (UGC). Hence statement 1 is correct.
• It enjoys the academic status and privileges of a university.
• It can design its own syllabus and courses to prepare the students to foray into diverse
fields after the completion of their studies. Hence statement 2 is incorrect.
• Deemed to be universities like other universities have the autonomy of offering various
courses and therefore, preparing students for various examinations and awards.

Source: https://www.livemint.com/news/india/ncert-gets-deemed-university-status-
announces-education-minister-dharmendra-pradhan-11693564926033.html

Q.7) Solution (a)

https://t.me/visioniastestseries2024
https://upscmaterial.online/

IASBABA’S PRELIMS TEST SERIES (AIPTS) 2024 – TEST 10 (CURRENT AFFAIRS – SEPT ’23)

Statement Analysis:

• The Constitution specifies that six months should not elapse between two
parliamentary sessions.
• This provision was borrowed from the Government of India Act of 1935. Hence
statement 1 is correct.
• India’s Parliament has no fixed calendar of sittings and the central government
determines the date and duration of parliamentary sessions.
• The Cabinet Committee on Parliamentary Affairs takes this decision. It currently has
ten Ministers, including those for Defence, Home, Finance, Agriculture, Tribal Affairs,
Parliamentary Affairs, and Information and Broadcasting. The Law Minister and the
Minister of State for External Affairs are special invitees to the Committee. Hence
statement 2 is incorrect.

Note: The Constitution does not use the term “special session”.The term sometimes refers to
sessions the government has convened for specific occasions like commemorating
parliamentary or national milestones. However, Article 352 (Proclamation of Emergency) of
the Constitution does refer to a “special sitting of the House”.

Source: https://indianexpress.com/article/explained/explained-politics/special-session-of-
parliament-8920401/

Q.8) Solution (c)

Statement Analysis:

• Walking leaves are known for their unusual appearance as they look similar to parts of
plants such as twigs, bark, or in the case of leaf insects as leaves. Hence statement 1
is correct.
• They are part of the Phasmatodea order and are also referred to as “phasmids” or
“stick and leaf insects”.
• They feed on plants and typically inhabit densely vegetated areas. Hence statement 2
is correct.
• Their natural range extends from islands in the Indian Ocean, across parts of mainland
South Asia and Southeast Asia, to Papua New Guinea and Australia in the western
Pacific.
• They are also largely nocturnal, resulting in a relative lack of movement during the day
that makes locating them quite difficult in the wild.

https://t.me/visioniastestseries2024
https://upscmaterial.online/

IASBABA’S PRELIMS TEST SERIES (AIPTS) 2024 – TEST 10 (CURRENT AFFAIRS – SEPT ’23)

Source: https://theprint.in/science/researchers-finds-seven-new-species-of-leaf-
insects/1736769/

Q.9) Solution (b)

Statement Analysis:

The reasons for the declining population of penguins:


• Global warming causing early sea ice melting in Antarctica. Hence statement 1 is
correct.
• Unstable sea ice causes problems in breeding and raising their offspring as Emperor
penguins need stable sea ice to breed and raise their chicks. Hence statement 2 is
correct.
• Melting ice leads to premature exposure of chicks to water resulting in their drowning.
• The decreasing krill production because of climate change as krill provide food for fish,
seals, whales, and penguins. Hence statement 3 is incorrect.

Source: https://www.livemint.com/opinion/online-views/unabated-global-warming-will-
wipe-penguins-off-this-planet-
11693735403149.html#:~:text=The%20penguins'%20failed%20breeding%2C%20published,t
o%20hold%20through%20polar%20winters

Q.10) Solution (c)

Statement Analysis:

• Digital Infrastructure for Knowledge Sharing (DIKSHA) Platform provides quality e-


content for school education in States/UTs. Hence statement 1 is correct.
• It is an initiative of the Ministry of Education for digital online education. Hence
statement 2 is correct.
• It has embedded assistive technologies for learners with visual or hearing
impairments. Hence statement 3 is correct.
• It is yet to integrate personalized adaptive learning(PAL) software to cater to the
specific learning needs of students. Hence statement 4 is incorrect.
• Personalized Adaptive Learning(PAL) is a software-based approach that will allow each
student to have an individualized learning experience over the course of the
curriculum based on their unique needs and abilities.

Source: https://www.thehindu.com/education/centres-digital-infrastructure-for-knowledge-
sharing-education-platform-to-offer-ai-help/article67263862.ece

https://t.me/visioniastestseries2024
https://upscmaterial.online/

IASBABA’S PRELIMS TEST SERIES (AIPTS) 2024 – TEST 10 (CURRENT AFFAIRS – SEPT ’23)

Q.11) Solution (b)

Statement Analysis:

• Aarogya Maitri Cube is the world’s first disaster hospital that can be airlifted. Hence
statement 1 is incorrect.
• It has been developed indigenously as part of India’s Aarogya Maitri initiative to
provide essential medical supplies to friendly countries. Hence statement 2 is correct.
• It has been developed indigenously under the Project Bharat Health Initiative for
Sahyog Hita and Maitri (BHISM).
• It is packed in 72 cubes that contain essential equipment and supplies, such as an
operation theatre, a mini-ICU, ventilators, blood test equipment, an X-ray machine, a
cooking station, food, water, shelter, a power generator, and more. Hence statement
3 is correct.

Source: https://www.timesnownews.com/india/72-cubes-200-survivors-alive-for-48-hours-
india-builds-worlds-first-portable-hospital-article-
103329095#:~:text=India%20has%20built%20the%20world's,during%20natural%20disasters
%20or%20crises

Q.12) Solution (a)

Statement Analysis:

• Poshan 2.0 is a centrally sponsored scheme being implemented through the State
government/ UT administrations based on a cost-sharing ratio between the Central
and the State Governments.
• It addresses malnutrition among children up to 6 years of age, adolescent girls, and
pregnant and lactating women. Hence statement 1 is incorrect.
• It aims to create an integrated nutrition support program that enhances the content,
delivery, outreach, and overall outcomes of nutritional services.
• The 6th Rashtriya Poshan Maah is celebrated by the Ministry of Women and Child
Development (MoWCD) TO comprehensively address malnutrition using a life-cycle
approach. Hence statement 2 is incorrect.
• The main objective is to raise awareness about critical life stages, including pregnancy,
infancy, childhood, and adolescence, to promote better nutrition across India.
• The theme of the 6th Rashtriya Poshan Maah of 2023 is ‘Suposhit Bharat, Sakshar
Bharat, Sashakt Bharat’. It places importance on nutrition, education, and
empowerment in building a healthier and stronger country. Hence statement 3 is
correct.

https://t.me/visioniastestseries2024
https://upscmaterial.online/

IASBABA’S PRELIMS TEST SERIES (AIPTS) 2024 – TEST 10 (CURRENT AFFAIRS – SEPT ’23)

Source: https://pib.gov.in/PressReleaseIframePage.aspx?PRID=1953762

Q.13) Solution (a)

Statement Analysis:

• Fiscal Deficit is the difference between the government's total expenditure and
its total revenue excluding borrowings. Hence statement 1 is incorrect.
• It is an indicator of the extent to which the government must borrow to finance its
operations and is expressed as a percentage of the country's GDP.
• A high fiscal deficit represents a debt burden while a low fiscal deficit represents a
healthy economy. Hence statement 2 is incorrect.
• It enables the government to increase spending on public services that can
stimulate economic growth.
• It can finance long-term investments, such as infrastructure projects, through fiscal
deficit.
• It can lead to job creation, which can help reduce unemployment and increase the
standard of living. Hence statement 3 is correct.

Source: https://www.thehindu.com/business/fiscal-deficit-touches-339-of-full-year-target-
at-end-july-cga-
data/article67256334.ece#:~:text=In%20absolute%20terms%2C%20the%20fiscal,General%2
0of%20Accounts%20(CGA)

Q.14) Solution (b)

Statement Analysis:

• Gabon is the 4th largest oil producer in Sub-Saharan Africa. Hence statement 1 is
incorrect.
• It is surrounded by the Atlantic Ocean in the west. It was a former French colony.
• Its capital is Libreville and shares land borders with Cameroon, Equatorial Guinea, and
the Republic of Congo.
• It is a member of the Organization of the Petroleum Exporting Countries (OPEC).
Hence statement 2 is correct.
• The Loango National Park is one of the country’s best wildlife-watching spots. Hence
statement 3 is correct.

https://t.me/visioniastestseries2024
https://upscmaterial.online/

IASBABA’S PRELIMS TEST SERIES (AIPTS) 2024 – TEST 10 (CURRENT AFFAIRS – SEPT ’23)

Source: https://www.thehindu.com/news/international/gabon-military-officers-seize-power-
days-after-presidential-election/article67251047.ece

Q.15) Solution (b)

Statement Analysis:

The ‘State of Food Security and Nutrition in the World (SOFI) 2023’, a report released by
the United Nations Food and Agriculture Organization (FAO). Some of its findings are:
• While global hunger numbers have remained stagnant between 2021 and 2022, the
number of people facing hunger worldwide has increased by over 122 million since
2019 due to the pandemic, repeated weather shocks, and conflicts, including the war
in Ukraine. Hence statement 1 is incorrect.
• Child malnutrition is high as 22.3% of children were stunted, 6.8% were wasted, and
5.6% were overweight in 2021. Hence statement 2 is correct.
• South Asia recorded the highest number of individuals unable to afford a healthy diet.
Hence statement 3 is correct.
• India has the lowest cost of a healthy diet among BRICS nations and its neighbours. In
2021, a healthy diet in India costs approximately 3.066 Purchasing Power Parity
(PPP) per person per day. Hence statement 4 is incorrect.

Source: https://epaper.thehindu.com/ccidist-
ws/th/th_delhi/issues/49910/OPS/GBDBMHI0J.1+G39BMIAG7.1.html

https://t.me/visioniastestseries2024
https://upscmaterial.online/

IASBABA’S PRELIMS TEST SERIES (AIPTS) 2024 – TEST 10 (CURRENT AFFAIRS – SEPT ’23)

Q.16) Solution (c)

Statement Analysis:

The Air Quality Life Index is a pollution index that quantifies the relationship between long-
term exposure to particulate pollution and life expectancy.
The key findings of the Air Quality Life Index released by the Energy Policy Institute at the
University of Chicago (EPIC) are:
• It mentions pollution as the foremost external threat to human health around the
world. Hence statement 1 is correct.
• Its impact on life expectancy rivals that of smoking, surpassing alcohol use, unsafe
water, and even exceeding car crash injuries.
• Bangladesh, India, Nepal, and Pakistan are the four most polluted countries in the
world. Hence statement 2 is correct.
• According to this index, Delhi is the world’s most polluted city. Hence statement 3 is
correct.
• Northern plains are the most polluted region. It includes the states and Union
Territories of Bihar, Chandigarh, Delhi, Haryana, Punjab, Uttar Pradesh, and West
Bengal.

Source: https://www.thehindu.com/data/the-northern-plains-of-india-is-the-most-polluted-
region-
data/article67280174.ece#:~:text=The%20northern%20plains%2C%20home%20to,Uttar%20
Pradesh%2C%20and%20West%20Bengal

Q.17) Solution (b)

Statement Analysis:

• Pulikkali is one of the folk art forms of Kerala. Hence statement 1 is incorrect.
• The artists wear tiger masks, paint their bodies like tigers, and dance to the rhythm of
traditional percussion instruments such as thakil, udukku, and chenda.
• The main theme of this folk art is tiger hunting. Hence statement 2 is correct.
• It is performed on the fourth day of the Onam festival. Hence statement 3 is correct.
• It is believed to have originated two centuries ago as a street dance form and became
part of Onam celebrations later.
• It is said to have been introduced by the erstwhile ruler of Cochin, Maharaja Rama
Varma Sakthan Thampuran.

Source: https://www.thehindu.com/news/national/kerala/thrissur-rocks-to-the-rhythm-of-
pulikkali/article67260629.ece#:~:text=Thrissur%20city%20turns%20into%20a,carnivalesque

https://t.me/visioniastestseries2024
https://upscmaterial.online/

IASBABA’S PRELIMS TEST SERIES (AIPTS) 2024 – TEST 10 (CURRENT AFFAIRS – SEPT ’23)

%20finish%20to%20Onam%20celebrations&text=Drumbeats%20and%20exciting%20roars%
20of,mood%20was%20wild%20and%20electric

Q.18) Solution (d)

Statement Analysis:

The key features of the National Education Policy 2020:


• Its primary objective is to prioritize learning in a student’s mother tongue. Hence
statement 1 is correct.
• It aims to create the National Research Foundation to facilitate research within
academic institutions. Hence statement 2 is correct.
• It establishes the National Curriculum Framework for School Education to set specific
learning standards. Hence statement 3 is correct.
• It establishes a National Credit Framework to encompass various academic practices
within a credit bank. Hence statement 4 is correct.

Source: https://timesofindia.indiatimes.com/blogs/toi-edit-page/indias-talent-pool-a-force-
for-national-global-good/

Q.19) Solution (c)

Statement Analysis:

• The Gujarat Declaration was adopted at the first World Health Organization (WHO)
Traditional Medicine Global Summit. Hence statement 1 is correct.
• The first World Health Organization (WHO) Traditional Medicine Global Summit was
held in Gandhinagar, Gujarat.
• It placed importance on global commitments towards indigenous knowledge,
biodiversity, and traditional, complementary, and integrative medicine. Hence
statement 2 is correct.

Source: https://pib.gov.in/PressReleaseIframePage.aspx?PRID=1954558

Q.20) Solution (b)

Statement Analysis:

• The Pradhan Mantri Dakshata Aur Kushalata Sampanna Hitgrahi(PM-DAKSH) Yojana is


a central sector scheme launched by the Ministry of Social Justice & Empowerment
(MoSJ&E). Hence statement 1 is correct.

https://t.me/visioniastestseries2024
https://upscmaterial.online/

IASBABA’S PRELIMS TEST SERIES (AIPTS) 2024 – TEST 10 (CURRENT AFFAIRS – SEPT ’23)

• It aims to enhance the competency level of the target groups to make them
employable both in self-employment and wage-employment for their socio-economic
development.
• There is no income limit for SCs, SafaiKaramcharis including waste picker and DNT.
• The annual family income should be below Rs.3 lakh for OBCs and EBCs (Economically
Backward Classes), and the annual family income should be below Rs.1 lakh. Hence
statement 2 is incorrect.
• The age criterion for the scheme is between 18-45 years. Hence statement 3 is correct.
• The stipend under this scheme is Rs.1,500/- per month to SCs and Safai Karamcharis
and Rs.1,000/- per month to OBCs/EBCs/DNTs for non-residential short-term and
long-term training courses. Hence statement 4 is incorrect.

Source: https://pib.gov.in/PressReleasePage.aspx?PRID=1954579

Q.21) Solution (b)

Statement Analysis:

• The Education to Entrepreneurship Initiative is a collaboration between the Ministry


of Education, the Ministry of Skill Development & Entrepreneurship, and Meta. Hence
statement 1 is incorrect.
• It aims to provide digital marketing skills training to five lakh entrepreneurs over the
next three years. Hence statement 2 is correct.
• Under this initiative, training will be conducted in seven regional languages by using
platforms like Facebook, WhatsApp, and Instagram. Hence statement 3 is correct.
• It will equip India’s talent pool with futuristic technologies, fostering a new generation
of problem solvers and entrepreneurs.

Source: https://pib.gov.in/PressReleaseIframePage.aspx?PRID=1954601

Q.22) Solution (c)

Statement Analysis:

• The Kākāpō is a species of parrot found only in New Zealand. Hence statement 1 is
incorrect.
• It is listed as critically endangered on the IUCN List. Hence statement 2 is correct.
• It is the world’s only flightless parrot. Hence statement 3 is correct.
• It is the world’s heaviest parrot. Hence statement 4 is correct.
• It is nocturnal and herbivorous.

https://t.me/visioniastestseries2024
https://upscmaterial.online/

IASBABA’S PRELIMS TEST SERIES (AIPTS) 2024 – TEST 10 (CURRENT AFFAIRS – SEPT ’23)

• It is also possibly one of the world’s longest-living birds, with a reported lifespan of up
to 100 years.
• They only breed every few years, triggered by the availability of certain forest foods
such as the fruits of the native rimu tree.
• It is also the only parrot to have a polygynous lek breeding system.

Note: Lek breeding system is a mating system in which the male provides no parental care to
its offspring.
Source: https://www.downtoearth.org.in/blog/wildlife-biodiversity/how-gene-mapping-
almost-all-remaining-k-k-p-will-help-nz-s-rare-night-parrot-survive-91464

Q.23) Solution (a)

Statement Analysis:

• The G20 works in three major tracks. The official tracks are the Finance Track and the
Sherpa Track. The unofficial track includes engagement groups or civil society groups.
• The Finance Track is headed by the finance ministers and central bank governors and
mainly focuses on issues of the global economy.
• It mainly focuses on fiscal and monetary policy issues such as the global economy,
infrastructure, financial regulation, financial inclusion, international financial
architecture, and international taxation. Hence statement 1 is correct.
• The Sherpa Track is headed by representatives of heads of state and mainly focuses on
socio-economic issues.
• It mainly focuses on socio-economic issues such as agriculture, anti-corruption,
climate, digital economy, education, employment, energy, environment, health,
tourism, trade and investment.

https://t.me/visioniastestseries2024
https://upscmaterial.online/

IASBABA’S PRELIMS TEST SERIES (AIPTS) 2024 – TEST 10 (CURRENT AFFAIRS – SEPT ’23)

• The unofficial track comprises engagement or civil groups. These groups often draft
recommendations to the G20 Leaders that contribute to the policy-making process.
• The engagement groups are as follows: Business20, Civil20, Labour20, Parliament20,
Science20, SAI20, Startup20, Think20, Urban20, Women20 and Youth20. Hence
statement 2 is incorrect.

Source: https://indianexpress.com/article/explained/explained-global/g-20-workstreams-
sherpa-finance-track-8924358/

Q.24) Solution (d)

Statement Analysis:

• The Johannesburg Declaration and the Contingent Reserve Arrangement are related
to BRICS.
• The Johannesburg Declaration predominantly emphasizes intra-BRICS cooperation
and collaboration with other developing nations. The member nations have agreed to
promote the use of local currencies in trade and financial transactions between BRICS
countries and other trade partners.
• The Contingent Reserve Arrangement aids countries facing short-term balance-of-
payments challenges.
Hence option d is correct.

Source: https://www.thehindu.com/opinion/op-ed/the-implications-of-the-expansion-of-
brics/article67274512.ece

Q.25) Solution (b)

Statement Analysis:

• India is the world’s second-largest consumer and the world’s largest importer of
vegetable oil. Hence statement 1 is incorrect.
• India is one of the largest producers of oilseeds in the world.
• India produces- rapeseeds, soybean, peanut, groundnut, copra, cottonseeds and
sunflower seeds.
• In terms of volumes, palm oil, soya bean oil, and mustard oil are the three largest
consumed edible oils in India. Hence statement 2 is correct.
• Palm oil (Crude + Refined) constitutes roughly 62% of the total edible oils imported
and is imported mainly from Indonesia and Malaysia.
• Soyabean oil (22%) is imported from Argentina and Brazil.
• Sunflower oil (15%) is imported mainly from Ukraine and Russia.

https://t.me/visioniastestseries2024
https://upscmaterial.online/

IASBABA’S PRELIMS TEST SERIES (AIPTS) 2024 – TEST 10 (CURRENT AFFAIRS – SEPT ’23)

Source: https://www.downtoearth.org.in/news/agriculture/india-looking-at-record-edible-
oil-imports-august-2023-demand-35-higher-than-last-year-91761

Q.26) Solution (b)

Statement Analysis:

• Bhoj Wetland is a Ramsar site (2002) located in Madhya Pradesh. Hence statement 1
is incorrect.
• It consists of two contiguous human-made reservoirs. Hence statement 2 is correct.
• It has two lakes, Upper Lake, also called Bhojtal, and Lower Lake or Chhota Talaab.
• It was first conceived by King Paramara Raja Bhoj in 1005-1055 CE. Hence statement
3 is correct.
• He built the lake by raising an earthen dam across the Kolans.
• The Lower Lake was built much later in 1794 by Chhote Khan, a minister to Nawab
Hayath Mohammad Khan.
• Bhadbhada Dam was built on the southeast corner of Bhojtal in 1965.

Source: https://www.thehindu.com/news/national/very-drastic-damage-ngt-tells-mp-govt-
to-stop-cruise-motorised-boats-in-wetlands/article67300195.ece

Q.27) Solution (c)

Statement Analysis:

• Nipah Virus (NiV) is a zoonotic virus with its natural host being fruit bats. Hence
statement 1 is correct.
• A zoonotic virus is transmitted from animals to humans.
• Its incubation period is 4 to 14 days with a fatality rate of 40% to 75%.
• The incubation period is the interval from infection to the onset of symptoms.
• Human infections range from asymptomatic infection to acute respiratory infection
(mild, severe), and fatal encephalitis.
• Infected people initially develop symptoms including fever, headaches, myalgia
(muscle pain), vomiting, and sore throat.
• This can be followed by dizziness, drowsiness, altered consciousness, and neurological
signs that indicate acute encephalitis.
• It is listed as a priority disease for the WHO Research and Development Blueprint.
Hence statement 2 is correct.
• There are currently no drugs or vaccines specific to treat this infection. Hence
statement 3 is correct.

https://t.me/visioniastestseries2024
https://upscmaterial.online/

IASBABA’S PRELIMS TEST SERIES (AIPTS) 2024 – TEST 10 (CURRENT AFFAIRS – SEPT ’23)

IMAGE SOURCE: gmsciencein.com

Source: https://www.downtoearth.org.in/news/health/nipah-outbreak-questions-being-
asked-about-kerala-s-public-health-system-to-detect-exotic-diseases-91727

Q.28) Solution (b)

Statement Analysis:

• The Northern Sea Route (NSR) connects the eastern and western regions of the Arctic
Ocean. Hence statement 1 is incorrect.
• It passes through four seas of the Arctic Ocean.
• It is the shortest shipping route connecting Europe and the Asia-Pacific region. Hence
statement 2 is correct.
• The route between Europe and Asia is just 13,000 km long, compared to the 21,000
km covered by the Suez Canal route, which reduces the travel duration from one
month to less than two weeks.
• The route starts at the boundary between the Barents and Kara seas (Kara Strait) and
concludes at the Bering Strait (Provideniya Bay).

https://t.me/visioniastestseries2024
https://upscmaterial.online/

IASBABA’S PRELIMS TEST SERIES (AIPTS) 2024 – TEST 10 (CURRENT AFFAIRS – SEPT ’23)

IMAGE SOURCE: eusp.org


Source: https://www.thehindu.com/news/international/india-russia-exploring-use-of-
northern-sea-route-eastern-maritime-corridor/article67304566.ece

Q.29) Solution (c)

Statement Analysis:

Exquisite product State


1. Longpi Pottery Manipur
2. Sheep Wool Stoles Himachal Pradesh Hence option c is correct.
3. Meenakari Rajasthan • Longpi pottery is
associated with the Tangkhul Naga
4. Araku Valley Coffee Andhra Pradesh tribes. Unlike most pottery,
Longpi does not resort to the
potter’s wheel.
• Sheep Wool Stoles are associated with the Bodh, Bhutia, and Gujjar Bakarwal tribes.
They exhibit their ingenuity with pure sheep wool, fashioning a diverse range of
apparel, from jackets to shawls and stoles
• Meenakari is the art of decorating metal surfaces with vibrant mineral substances, a
technique introduced by the Mughals.
• Araku Valley Coffee is renowned for its unique flavors and sustainable cultivation
practices.

Source: https://pib.gov.in/PressReleasePage.aspx?PRID=1956343

https://t.me/visioniastestseries2024
https://upscmaterial.online/

IASBABA’S PRELIMS TEST SERIES (AIPTS) 2024 – TEST 10 (CURRENT AFFAIRS – SEPT ’23)

Q.30) Solution (d)

Statement Analysis:

Morocco is the only African country with coastal exposure to both the Atlantic Ocean and the
Mediterranean Sea. It lies directly across the Strait of Gibraltar from Spain. Its national
language is Arabic. It has a Mediterranean climate, with mild wet winters and hot dry
summers. It has Algeria to the east and southeast, Western Sahara to the south, the Atlantic
Ocean to the west, and the Mediterranean Sea to the north. Hence option d is correct.

IMAGE SOURCE: WorldAtlas

Source: https://indianexpress.com/article/explained/explained-global/earthquake-morocco-
8932001/

Q.31) Solution (c)

Statement Analysis:

• The Central Bank Digital Currency (CBDC) is fiat money backed by a government

https://t.me/visioniastestseries2024
https://upscmaterial.online/

IASBABA’S PRELIMS TEST SERIES (AIPTS) 2024 – TEST 10 (CURRENT AFFAIRS – SEPT ’23)

through its central bank in a virtual form. Hence statement 1 is correct.


• It combines the power of blockchain with the logistics of distributed ledger technology
(DLT), where data can be synchronized across multiple locations without the need for
centralized storage. Hence statement 2 is correct.
• Just like paper currency, each unit is uniquely identifiable to prevent counterfeiting.
Hence statement 3 is correct.
• It is a liability of the central bank just as physical currency is.
• It’s a digital bearer instrument that can be stored, transferred, and transmitted by all
kinds of digital payment systems and services.

Source: https://indianexpress.com/article/business/banking-and-finance/rbi-banks-plan-
new-features-boost-digital-currency-transactions-8934317/

Q.32) Solution (b)

Statement Analysis:

• Recorded Forest Area (RFA) refers to lands that are recorded as forests, regardless of
their official classification or ownership. Hence statement 1 is correct.
• It includes notified forests and unclassed forests. Hence statement 2 is incorrect.
• Unclassed forests are forests not officially categorized.
• Over half of the Northeast region is categorized as RFA. Hence statement 3 is correct.

Source: https://www.thehindu.com/sci-tech/energy-and-environment/forest-conservation-
amendment-act-threat-northeast-states/article67268866.ece

Q.33) Solution (a)

Statement Analysis:

• Several banks have enabled the interoperability of the Unified Payments Interface’s
(UPI) Quick Response (QR) code with their Central Bank Digital Currency(CBDC). Hence
statement 1 is correct.
• Interoperability of UPI with the digital rupee means all UPI QR codes are compatible
with CBDC apps.
• A QR code consists of black squares arranged in a square grid on a white background,
which can be read by an imaging device such as a camera. It contains information
about the item to which it is attached.
• It ensures seamless transactions between a customer and merchant without needing
to switch between multiple digital platforms. Hence statement 2 is correct.

https://t.me/visioniastestseries2024
https://upscmaterial.online/

IASBABA’S PRELIMS TEST SERIES (AIPTS) 2024 – TEST 10 (CURRENT AFFAIRS – SEPT ’23)

• It will allow a digital rupee user to make payments for their daily needs, such as
groceries and medicines, by scanning any UPI QR codes at any merchant outlet.
Source: https://indianexpress.com/article/explained/explained-economics/upi-qr-code-cdbc-
banks-explained-e-rupee-8925038/

Q.34) Solution (c)

Statement Analysis:

• The “Assessment Report on Invasive Alien Species and their Control’’ released by the
Intergovernmental Platform on Biodiversity and Ecosystem Services (IPBES) reports:
• The water hyacinth is the world’s most widespread invasive alien species on
land. Hence statement 1 is correct.
• There are 37,000 alien species introduced by human activities worldwide.
• Over 3,500 of these alien species pose significant global threats to nature, the
economy, food security, and human health.
• Invasive alien species contribute to 60% of global plant and animal extinctions. Hence
statement 2 is correct.
• The annual costs of invasive alien species have quadrupled every decade since 1970.
Hence statement 3 is correct.
• Invasive alien species like Aedes albopictus and Aedes aegyptii spread diseases such
as malaria, Zika, and West Nile Fever.
• Invasive alien species can also amplify the impacts of climate change. For example,
invasive alien plants, especially trees and grasses, can sometimes be highly flammable
and promote more intense fires.

Source: https://indianexpress.com/article/india/biodiversity-loss-37000-alien-species-
introduced-by-human-activities-says-report-8924756/

Q.35) Solution (a)

Statement Analysis:

• Battery storage or battery energy storage systems (BESS) are devices that enable
energy from renewables to be stored and then released when the power is needed
most. Hence statement 1 is correct.
• Viability Gap Finance means a grant to support projects that are economically justified
but not financially viable. Viability Gap Funding is provided as a grant one-time or
deferred to make a project commercially viable.

https://t.me/visioniastestseries2024
https://upscmaterial.online/

IASBABA’S PRELIMS TEST SERIES (AIPTS) 2024 – TEST 10 (CURRENT AFFAIRS – SEPT ’23)

• The Scheme for Viability Gap Funding (VGF) for the development of Battery Energy
Storage Systems (BESS) envisages the development of 4,000 MWh of BESS projects by
2030-31. Hence statement 2 is incorrect.
• It provides financial support up to 40% of the capital cost will be supported in the form
of Viability Gap Funding (VGF). The entire viability gap funding (VGF) will be borne by
the Central Government.
• The VGF shall be disbursed in 5 tranches linked with the various stages of
implementation of BESS projects.
• To ensure that the benefits of the scheme reach the consumers, a minimum of 85% of
the BESS project capacity will be made available to Distribution Companies (Discoms).

Source:
https://pib.gov.in/PressReleaseIframePage.aspx?PRID=1955113#:~:text=The%20VGF%20for
%20development%20of,(LCoS)%20ranging%20from%20Rs

Q.36) Solution (d)

Statement Analysis:

The East Asia Summit (EAS) is the Indo-Pacific’s premier forum for strategic dialogue. It is the
only leader-led forum at which all key Indo-Pacific partners meet to discuss political, security,
and economic challenges facing the region and has an important role to play in advancing
closer regional cooperation. It comprises eighteen participating countries including ten ASEAN
Member States (Brunei Darussalam, Cambodia, Indonesia, Lao PDR, Malaysia, Myanmar,
Philippines, Singapore, Thailand, Vietnam) and Australia, China, India, Japan, New Zealand,
the Republic of Korea, the Russian Federation and the United States of America. Hence option
d is correct.
Source: https://www.thehindu.com/news/national/comprehensive-strategic-partnership-
injected-new-dynamism-in-ties-with-asean-pm-modi/article67277674.ece

Q.37) Solution (a)

Statement Analysis:

• The Malaviya Mission is a teachers' training programme by the University Grants


Commission. Hence statement 1 is correct.
• It aims to improve the capacity building of faculty members in higher educational
institutions (HEIs). Hence statement 2 is incorrect.
• It improves the quality of teachers’ training, builds leadership skills in teachers, and
helps realize the goals of the National Education Policy (NEP).

https://t.me/visioniastestseries2024
https://upscmaterial.online/

IASBABA’S PRELIMS TEST SERIES (AIPTS) 2024 – TEST 10 (CURRENT AFFAIRS – SEPT ’23)

• It will ensure continuous professional development and help in building capacities of


15 lakh teachers of HEIs through 111 Malaviya Mission Centers (formerly known as
Human Resource Development Centers (HRDCs)). The capacity building under the
Mission will be mapped to the credit framework to ensure career progression
pathways for educators.
• The themes identified for the course curriculum for Capacity building of Faculty
Members in HEIs include - Multidisciplinary Education, Indian Knowledge Systems
(IKS), Academic Leadership, Research and Development, Skill Development, and
Student Diversity among others.
Source: https://pib.gov.in/PressReleaseIframePage.aspx?PRID=1954967

Q.38) Solution (b)

Statement Analysis:

• The Blue Button Movement is organised by the Dementia India Alliance (DIA) on
September 21 to mark World Alzheimer’s Day. Hence statement 1 is incorrect.
• The blue button is a symbol of awareness and support for people affected by dementia.
• This campaign will include the formation of a human chain symbolically resembling
the blue button like a shield or a coat of arms, unifying and encouraging people to
recognize the impact of dementia, empathize, and take proactive steps to support
patients.
• DemClinic will be the first national comprehensive expert-led cognitive assessment
platform for the elderly. Hence statement 2 is correct.
• It is aimed at supporting families and persons with dementia through post-diagnostic
care and support.

Source: https://www.thehindu.com/sci-tech/health/free-dementia-support-line-and-online-
memory-clinic-launched-in-india-blue-
button/article67273028.ece#:~:text=Those%20seeking%20guidance%2C%20support%2C%2
0or,can%20call%208585%20990%20990

Q.39) Solution (c)

Statement Analysis:

The Global Biofuel Alliance (GBA) is backed by Brazil, India, and the USA. It aims to facilitate
cooperation and intensify the use of sustainable biofuels, including in the transportation
sector. It will place emphasis on strengthening markets, facilitating global biofuels trade,
developing concrete policy lesson-sharing, and providing technical support for national
biofuels programs worldwide. Hence option c is correct.

https://t.me/visioniastestseries2024
https://upscmaterial.online/

IASBABA’S PRELIMS TEST SERIES (AIPTS) 2024 – TEST 10 (CURRENT AFFAIRS – SEPT ’23)

Source:
https://pib.gov.in/PressReleaseIframePage.aspx?PRID=1955836#:~:text=GBA%20as%20a%2
0tangible%20outcome,exporting%20technology%20and%20exporting%20equipment

Q.40) Solution (d)

Statement Analysis:

HII-A rocket to explore the origins of the universe is launched by Japan. The rocket put a
satellite called X-Ray Imaging and Spectroscopy Mission, or XRISM, into orbit around Earth. It
will measure the speed and makeup of what lies between galaxies. This information will help
in studying how celestial objects were formed. It can also lead to solving the mystery of how
the universe was created. Hence option d is correct.

Source: https://www.thehindu.com/sci-tech/technology/japan-launches-rocket-lunar-lander-
xray-telescope-explore-origins-universe/article67279747.ece/amp/

Q.41) Solution (c)

Statement Analysis:

• The Konark Sun Temple was constructed during the reign of King Narasimhadeva I of
the Eastern Ganga Dynasty. Hence statement 1 is correct.
• It was constructed in the 13th century. It is designed in the shape of a colossal
chariot with 12 pairs of stone-carved wheels. They represent the chariot of the Sun
God.
• It was built using Khondalite rocks and was declared a UNESCO World Heritage Site in
1984. Hence statement 2 is correct.

Source: https://indianexpress.com/article/india/g20-summit-konark-wheel-odisha-
handshake-modi-8931543/

Q.42) Solution (b)

Statement Analysis:

Amitava Roy Committee formed by the Supreme Court of India in 2018 is related to Prison
Reforms. Its objective is to examine the various problems plaguing prisons in the country,
from overcrowding to lack of legal advice to convicts to issues of remission and parole. Hence
option b is correct.

https://t.me/visioniastestseries2024
https://upscmaterial.online/

IASBABA’S PRELIMS TEST SERIES (AIPTS) 2024 – TEST 10 (CURRENT AFFAIRS – SEPT ’23)

Source: https://indianexpress.com/article/india/denied-basic-facilities-women-prisoners-
impacted-worse-men-sc-panel-prison-reforms-8925934/

Q.43) Solution (d)

Statement Analysis:

PM Gati Shakti plan envisages a centralised portal to unite the infrastructural initiatives
planned and initiated by as many as 16 central ministries and departments.

• GatiShakti targets to cut logistic costs, increase cargo handling capacity and reduce
the turnaround time.
• It is a campaign to lend more speed (Gati) and power (Shakti) to projects by connecting
all concerned departments on one platform.
• This way, the infrastructure schemes of various ministries and state governments will
be designed and executed with a common vision.

The objectives of PM Gati Shakti are:


• Establish synergies among different ministries during the planning stage itself
• Align timelines of various projects through efficient planning
• Promote standardization of the designs for efficient project approval
• Enable standardized procedures by all the agencies to reduce project delays
Hence option d is correct.

Source:
https://pib.gov.in/PressReleaseIframePage.aspx?PRID=1955460#:~:text=India%20showcased
%20PM%20Gatishakti%20National,ADB)%20at%20Tbilisi%2C%20Georgia

Q.44) Solution (c)

Statement Analysis:

Recently, a white sambar was photographed in the Cauvery Wildlife Sanctuary which was
constituted in 1989 under the Wildlife Protection Act of 1972. It is located in Karnataka. It
consists of reserve forests in Chamarajnagar, Ramanagar, and Mandya Districts of Karnataka
State. It has dry deciduous forests, scrublands, grasslands, and riverine ecosystems. Hence
option c is correct.

Source: https://www.thehindu.com/news/national/karnataka/researchers-discover-white-
sambar-in-cauvery-wildlife-sanctuary/article67276939.ece

https://t.me/visioniastestseries2024
https://upscmaterial.online/

IASBABA’S PRELIMS TEST SERIES (AIPTS) 2024 – TEST 10 (CURRENT AFFAIRS – SEPT ’23)

Q.45) Solution (d)

Statement Analysis:

• Basket Sneaking, False Urgency, Subscription Trap, and Nagging are the terms related
to dark patterns. Hence option d is correct.
• Dark Patterns are deceptive design patterns using user interface or user experience
interactions on any platform. These practices are designed to mislead or trick users
into doing something they originally did not intend or want to do. It undermines
consumer autonomy, decision-making, and choice potentially constituting misleading
advertising, unfair trade practices, or violations of consumer rights.

Source: https://newsonair.gov.in/Main-News-
Details.aspx?id=467407#:~:text=Centre%20has%20sought%20public%20comments,choice%
20and%20violate%20consumer%20rights

Q.46) Solution (b)

Statement Analysis:

• The Incremental cash reserve ratio (I-CRR) is an additional cash balance that the
Reserve Bank of India can ask banks to maintain over and above the Cash Reserve
Ratio(CRR). Hence statement 1 is incorrect.
• It was introduced on August 10, 2023, by RBI as the level of surplus liquidity in the
system surged because of the return of Rs 2,000 banknotes to the banking system.
• It was announced as a temporary measure to absorb excess liquidity from the banking
system. Hence statement 2 is correct.

Source: https://www.thehindu.com/business/Economy/rbi-decides-to-discontinue-i-crr-in-a-
phased-
manner/article67284514.ece#:~:text=While%2025%25%20of%20the%20I,released%20on%2
0October%207%2C%202023&text=The%20Reserve%20Bank%20of%20India,CRR)%20in%20
a%20phased%20manner

Q.47) Solution (c)

Statement Analysis:

• Hallmarking is the accurate determination and official recording of the proportionate


content of precious metals in precious metal articles. Hence statement 1 is correct.

https://t.me/visioniastestseries2024
https://upscmaterial.online/

IASBABA’S PRELIMS TEST SERIES (AIPTS) 2024 – TEST 10 (CURRENT AFFAIRS – SEPT ’23)

• It is the official mark used in many countries as a guarantee of purity or fineness of


precious metal articles.
• The objectives of the Hallmarking Scheme are to protect the public against
adulteration and to obligate manufacturers to maintain legal standards of fineness.
• At present, only two precious metals – gold and silver come under hallmarking in India.
Hence statement 2 is correct.

Source:
https://pib.gov.in/PressReleaseIframePage.aspx?PRID=1955517#:~:text=The%20third%20ph
ase%20of%20the%20mandatory%20hallmarking%20will%20cover%20additional,of%20distri
cts%20covered%20under%20mandatory

Q.48) Solution (c)

Statement Analysis:

• ‘Bharat: The Mother of Democracy’ Portal was launched by the Ministry of Culture on
the eve of the G20 leadership summit. Hence statement 1 is correct.
• It offers content that delves into India’s democratic legacy spanning an astonishing
7,000 years. Hence statement 2 is correct.
• The portal is divided into five sections and 22 sub-sections starting from Sindhu-
Saraswati Civilisation (6000- 2000 BCE), Mahajanapada and Gantantra (7-8 BCE),
Vijaynagar Empire (14-16 century) and Mughal emperor Akbar reign (1556-1605) to
Constitution of India (1947), and Elections in Modern India (1952 onwards).
• The content on the portal is available in 16 languages including German, French,
Spanish, and Arabic. Hence statement 3 is correct.
• The content for the portal has been curated by the Indira Gandhi National Centre for
the Arts (IGNCA), which is a center for research, academic pursuit, and dissemination
in the field of the arts under the Ministry of Culture.

Source: https://www.newindianexpress.com/nation/2023/sep/08/bharat-mother-of-
democracy-portal-launched-atg20-exhibition-2613009.html

Q.49) Solution (b)

Statement Analysis:

• Uttarakhand became the first State in the country to legalize cannabis cultivation.
Hence statement 1 is incorrect.
• Narcotic Drugs and Psychotropic Substances Act, 1985 prohibits the sale and
production of cannabis resin and flowers.

https://t.me/visioniastestseries2024
https://upscmaterial.online/

IASBABA’S PRELIMS TEST SERIES (AIPTS) 2024 – TEST 10 (CURRENT AFFAIRS – SEPT ’23)

• But Section 10 of the Act empowers the States to make rules regarding the cultivation
of any cannabis plant, production, possession, transport, consumption, use and
purchase and sale, and consumption of cannabis.
• The States are empowered to permit, by general or special order, the cultivation of
hemp only for obtaining fiber or seeds or for horticultural purposes.
• Hemp is a botanical class of Cannabis sativa cultivars grown specifically for industrial
or medicinal use. Hence statement 2 is correct.

Source: https://www.thehindu.com/news/national/other-states/himachal-pradesh-inches-
closer-to-cannabis-
cultivation/article67291491.ece#:~:text=With%20the%20Himachal%20Pradesh%20governm
ent,towards%20the%20use%2Dabuse%20of

Q.50) Solution (a)

Statement Analysis:

• The hybrid nanoparticles made of gold and copper sulphide can kill cancer cells. Hence
statement 1 is correct.
• They can create heat and release toxic oxygen when illuminated with light, effectively
targeting and killing cancer cells. Hence statement 2 is correct.
• The photoacoustic property of the nanoparticles allows them to absorb light and
generate ultrasound waves, which can be used to detect cancer cells with high contrast
once the particles reach them.

Source: https://www.hindustantimes.com/india-news/iisc-develops-approach-to-destroy-
cancer-cells-
101694545200621.html#:~:text=Scientists%20have%20created%20hybrid%20nanoparticles,
those%20forming%20solid%20tumour%20masses

Q.51) Solution (b)


Statement Analysis:
• The National eVidhan Application (NeVA) objective is to make all legislatures digital
with the help of Information & Communication Technologies (ICT). Hence statement
1 is correct.
• It epitomizes the concept of ‘One Nation, One Application’, with a mission to
prioritize cloud-first and mobile-first approaches.
• It enables automation of the entire law-making process, tracking of decisions and
documents, and sharing of information.

https://t.me/visioniastestseries2024
https://upscmaterial.online/

IASBABA’S PRELIMS TEST SERIES (AIPTS) 2024 – TEST 10 (CURRENT AFFAIRS – SEPT ’23)

• The Ministry of Electronics and Information Technology (MietY) provides technical


assistance to the project.
• The Ministry of Parliamentary Affairs (MoPA) provides funding to the project. The
funding of NeVA is through the Central Sponsored Scheme - 60:40 for states; 90:10
for North East & and hilly States and 100% for UTs. Hence statement 2 is incorrect.
• It is the nodal ministry for its implementation in all 31 States/UTs with Legislatures.
• Himachal Pradesh was the first Digital Legislature in the country. Hence statement 3
is correct.

Source: https://www.thehindu.com/news/national/other-states/president-droupadi-
murmu-launches-gujarat-assemblys-digital-project/article67304011.ece

Q.52) Solution (d)


Statement Analysis:
• Probiotics are a combination of live beneficial bacteria and/or yeasts.
• Probiotic supplements are a way to add good bacteria to our bodies.
• A probiotic community is made up of things called Microbes.
• These microbes are a combination of:
• Bacteria
• Fungi (including yeasts)
• Viruses
• Protozoa

For a microbe to be called a probiotic, it must have several characteristics. These include
being able to:

• Be isolated from a human


• Survive in your intestine after ingestion (being eaten)
• Have a proven benefit
• Be safely consumed
• Help your body digest food
• Keep bad bacteria from getting out of control and making you sick
• Create vitamins
• Help support the cells that line your gut to prevent bad bacteria that you may have
consumed (through food or drinks) from entering your blood
• Break down and absorb medications

https://t.me/visioniastestseries2024
https://upscmaterial.online/

IASBABA’S PRELIMS TEST SERIES (AIPTS) 2024 – TEST 10 (CURRENT AFFAIRS – SEPT ’23)

Probiotic-rich foods:
• Yogurt
• Buttermilk
• Sourdough bread
• Cottage cheese
• Kombucha
• Tempeh
• Fermented pickles
Hence option d is correct.

Source: https://www.downtoearth.org.in/news/health/how-can-i-lower-my-cholesterol-do-
supplements-work-how-about-psyllium-or-probiotics--91777

Q.53) Solution (a)

Statement Analysis:

• The Unified Portal for Agricultural Statistics (UPAg) is an advanced agricultural data
management platform that provides standardized agricultural data and agricultural
data analysis. Hence statement 1 is correct.
• It is developed by the Department of Agriculture and Farmers Welfare.
• Because it provides real-time information on crop production, market trends, pricing,
and other vital agricultural data along with giving information on production trends,
trade correlations, and consumption patterns, aiding policymakers in making
informed decisions. Hence statement 2 is correct.
• It will generate granular production estimates with increased frequency, enhancing
the government’s ability to respond to agricultural crises swiftly.
• It will produce commodity profiles using algorithms, minimizing subjectivity and
providing users with comprehensive insights.

Source:
https://pib.gov.in/PressReleaseIframePage.aspx?PRID=1957771#:~:text=Member%20of%20
NITI%20Aayog%2C%20Professor,India's%20agriculture%20sector%20is%20facing

Q.54) Solution (c)

Statement Analysis:

Pradhan Mantri Matasya Sampada Yojna (PMMSY) aims to bring about the Blue
Revolution through sustainable and responsible development of the fisheries sector in
India. It was introduced as part of the ‘Atma Nirbhar Bharat’ package with an investment of

https://t.me/visioniastestseries2024
https://upscmaterial.online/

IASBABA’S PRELIMS TEST SERIES (AIPTS) 2024 – TEST 10 (CURRENT AFFAIRS – SEPT ’23)

Rs. 20,050 crores, the highest-ever investment in this sector. is being implemented in all
States and UTs for a period of 5 years from FY 2020-21 to FY 2024-25.

The objectives of the Pradhan Mantri Matasya Sampada Yojna (PMMSY):


• It harnesses fisheries potential in a sustainable, responsible, inclusive, and equitable
manner and provides for social, physical, and economic security for fishers and fish
farmers. Hence statement 1 is correct.
• It enhances fish production and productivity through expansion, intensification,
diversification, and productive utilization of land and water. Hence statement 2 is
correct.
• It modernizes and strengthens the value chain, post-harvest management, and
quality improvement. Hence statement 3 is correct.
• It helps in building a robust fisheries management and regulatory framework.
• It works to double fishers' and fish farmers’ incomes and generate meaningful
employment.
• It enhances the contribution of the fisheries sector to agricultural Gross Value Added
(GVA) and exports.

Note: Matsya Sampada Jagrukta Abhiyan is organized by the Department of Fisheries to


disseminate information and knowledge about 9 Years of achievements of the Government
of India by highlighting the success stories of the beneficiaries and reaching out to 2.8 crore
fish farmers and 3477 coastal villages.

Source:
https://pib.gov.in/PressReleaseIframePage.aspx?PRID=1957760#:~:text=Union%20Minister
%20Fisheries%2C%20Animal%20Husbandry,Matasya%20Sampada%20Yojna%20(PMMSY)
https://pmmsy.dof.gov.in/

Q.55) Solution (a)

Statement Analysis:

• Ozone Layer exists in the Stratosphere, between 10 km and 40 km above the Earth’s
surface. Without the Ozone layer, radiation from the sun would reach the earth
directly, having ill effects on human health, i.e., eye cataracts, skin cancer, etc., and
adverse impacts on agriculture, forestry, and marine life.
• World Ozone Day is celebrated on the 16th of September each year to commemorate
the signing of the Montreal Protocol. Hence statement 1 is incorrect.

https://t.me/visioniastestseries2024
https://upscmaterial.online/

IASBABA’S PRELIMS TEST SERIES (AIPTS) 2024 – TEST 10 (CURRENT AFFAIRS – SEPT ’23)

• It aims to spread awareness among people about the depletion of the Ozone Layer
and the measures taken/ to be taken to preserve it.
• The theme of World Ozone Day 2023 is “Montreal Protocol: fixing the ozone layer
and reducing climate change”.
• The Kigali Agreement aims to phase down hydrofluorocarbons (HFCs) by curbing
both their production and consumption.
• It aims to achieve over 80% reduction in HFC consumption by 2047 which will curb a
global increase of temperature by up to 0.5 degrees Celsius by the end of the
century. Hence statement 2 is correct.
• India has phased out chlorofluorocarbons, carbon tetrachloride, halons, methyl
bromide, and methyl chloroform for controlled uses is in line with the Montreal
Protocol phase-out schedule.
• India is yet to phase down hydrofluorocarbons (HFCs). As per the Kigali Agreement,
India will complete its phase down of production and consumption of
hydrofluorocarbons (HFCs) for controlled uses in 4 steps from 2032 onwards with
cumulative reduction of 10% in 2032, 20% in 2037, 30% in 2042, and 85% in 2047.
Hence statement 3 is incorrect.

Note: The Vienna Convention came into force in 1985 and is an international treaty on the
protection of the Ozone Layer. Under this convention, the Montreal Protocol came into force
in 1987 to repair the ozone layer to protect the earth’s Ozone Layer by phasing out
production and consumption of Ozone Depleting Substances for end applications.

Source: https://newsonair.gov.in/News?title=Environment-Ministry-celebrates-29th-World-
Ozone-
Day&id=467868#:~:text=This%20Day%20is%20celebrated%20on,on%20this%20day%20in%2
01987

Q.56) Solution (c)

Statement Analysis:

• Santiniketan was originally an ashram built by Debendranath Tagore where anyone,


irrespective of caste and creed, could come and spend time meditating on the one
Supreme God.
• Later, Rabindranath Tagore envisioned it as a university town and established a
school. It was called Brahmacharya Ashram and was later expanded into the Visva-
Bharati University. Hence statement 1 is correct.
• It does not represent the British colonial architectural orientations of the early 20th
century and European modernism.

https://t.me/visioniastestseries2024
https://upscmaterial.online/

IASBABA’S PRELIMS TEST SERIES (AIPTS) 2024 – TEST 10 (CURRENT AFFAIRS – SEPT ’23)

• It represents approaches toward pan-Asian modernity, drawing on ancient, medieval,


and folk traditions from across the region. Hence statement 2 is incorrect.
• It is adorned with splendid sculptures, frescoes, murals, and paintings created by
renowned artists such as Rabindranath Tagore, Nandalal Bose, and Ramkinkar Baij.
Hence statement 3 is correct.
• It has been added to the World Heritage Site List recognized by the United Nations
Educational, Scientific, and Cultural Organization (UNESCO). Hence statement 4 is
correct.

https://t.me/visioniastestseries2024
https://upscmaterial.online/

IASBABA’S PRELIMS TEST SERIES (AIPTS) 2024 – TEST 10 (CURRENT AFFAIRS – SEPT ’23)

Note:
• Visva-Bharati in Santiniketan is Bengal’s only central university.
• There are 42 properties inscribed on the World Heritage List in India. You can go
through the list at https://whc.unesco.org/en/statesparties/in

Source: https://indianexpress.com/article/india/tagores-home-santiniketan-added-to-
unesco-world-heritage-list-

https://t.me/visioniastestseries2024
https://upscmaterial.online/

IASBABA’S PRELIMS TEST SERIES (AIPTS) 2024 – TEST 10 (CURRENT AFFAIRS – SEPT ’23)

8944134/#:~:text=Bordering%20Bolpur%20city%20in%20the,of%20Nobel%20laureate%20R
abindranath%20Tagore.&text=Santiniketan%2C%20the%20home%20of%20late,world%20bo
dy%20announced%20on%20Sunday

Q.57) Solution (b)

Statement Analysis:

• Scrub typhus is a disease caused by a bacteria called Orientia tsutsugamushi. Hence


statement 1 is incorrect.
• It is a major public health threat in South and Southeast Asia. India is one of the
hotspots with at least 25% of the disease burden.
• It spreads to people through bites of infected chiggers (larval mites). Hence
statement 2 is correct.
• The most common symptoms of scrub typhus include fever, headache, body aches,
and sometimes rash.
• It will not spread from person to person.
• There is no vaccine available for this disease. Hence statement 3 is incorrect.
• It should be treated with the antibiotic doxycycline. It can be used in persons of any
age. Hence statement 4 is correct.

Note: Leptospirosis is a potentially fatal zoonotic bacterial disease caused by Leptospira


interrogans or Leptospira. The carriers of the disease can be either wild or domestic animals,
including rodents, cattle, pigs, and dogs. Its symptoms include high fever, headache, chills,
vomiting, red eyes, abdominal pain, rashes, and diarrhea.

Source: https://www.livemint.com/news/india/odisha-hit-by-two-major-disease-outbreaks-
all-you-need-to-know-about-scrub-typhus-leptospirosis-
11694742176318.html#:~:text=Odisha%20is%20reeling%20from%20two,had%20succumbe
d%20to%20the%20disease

Q.58) Solution (d)

Statement Analysis:

• The African cheetah is listed as vulnerable on the IUCN List. Hence statement 1 is
correct.
• The Asiatic cheetah is listed as critically endangered on the IUCN List. Hence
statement 2 is correct.
• African cheetahs are around 6,500-7,000 African cheetahs present in the wild
whereas Asiatic cheetahs are around 40-50 found only in Iran.

https://t.me/visioniastestseries2024
https://upscmaterial.online/

IASBABA’S PRELIMS TEST SERIES (AIPTS) 2024 – TEST 10 (CURRENT AFFAIRS – SEPT ’23)

• The Asiatic cheetah is smaller and paler than the African cheetah.
• The African cheetah is bigger as compared to the Asiatic cheetah. Hence statement 3
is correct.
• Both Asiatic cheetah and African cheetah are listed as Appendix I of CITES. This List
comprises migratory species that have been assessed as being in danger of
extinction throughout all or a significant portion of their range. Hence statement 4 is
correct.
Source:
https://pib.gov.in/PressReleaseIframePage.aspx?PRID=1958158#:~:text=Commemorating%2
0one%20year%20of%20successful%20implementation%20of%20Project%20Cheetah%20in
%20India&text=17th%20September%2C%202022%20marked,local%20extinction%20from%
20the%20country

Q.59) Solution (c)

Statement Analysis:

• India is a member of the International Organization of Legal Metrology (OIML),


an international standard-setting body that develops model regulations and
standards.
• OIML plays a crucial role in harmonising national laws and regulations on the
performance of measuring instruments like clinical thermometers, alcohol breath
analysers, radar speed measuring instruments, ship tanks found at ports, and petrol
dispensing units. Hence statement 1 is correct.
• World Metrology Day is celebrated in commemoration of the anniversary of the
signing of the Metre Convention in 1875.
• The Metric Convention of 1875, more formally known as the Metre Convention or
Treaty of the Metre, is an international treaty that was signed on 20 May 1875, in
Paris, France. It established the International System of Units (SI), which is the
modern form of the metric system. Hence statement 2 is correct.
• The theme for World Meteorological Day 2023 is “The Future of Weather, Climate,
and Water Across Generations”. The theme emphasizes the need to address the
challenges posed by climate change and to develop strategies to ensure sustainable
water and weather-related practices for future generations. Hence statement 3 is
correct.

Source: https://pib.gov.in/PressReleaseIframePage.aspx?PRID=1957429

Q.60) Solution (b)

Statement Analysis:

https://t.me/visioniastestseries2024
https://upscmaterial.online/

IASBABA’S PRELIMS TEST SERIES (AIPTS) 2024 – TEST 10 (CURRENT AFFAIRS – SEPT ’23)

• The Business Responsibility and Sustainability Reporting (BRSR) is a mandatory


disclosure mechanism for the top 1000 listed companies or businesses to report
their performance on environmental, social, and corporate governance (ESG)
parameters. Hence statement 1 is incorrect.
• ESG represents a set of guidelines that compel companies to adhere to improved
standards in their operations, encompassing better governance, ethical conduct,
environmentally sustainable practices, and social responsibility.
• It is rooted in the nine principles outlined in the National Guidelines for Responsible
Business Conduct (NGRBC). Hence statement 2 is correct.
• The nine principles are:
✓ Businesses should conduct and govern themselves with integrity, and in a manner
that is ethical, transparent, and accountable.
✓ Businesses should provide goods and services in a manner that is sustainable and
safe.
✓ Businesses should respect and promote the well-being of all employees, including
those in their value chains.
✓ Businesses should respect the interests of and be responsive to all their stakeholders.
✓ Businesses should respect and promote human rights.
✓ Businesses should respect and make efforts to protect and restore the environment.
✓ Businesses, when engaging in influencing public and regulatory policy, should do so
in a manner that is responsible and transparent.
✓ Businesses should promote inclusive growth and equitable development.
✓ Businesses should engage with and provide value to their consumers in a responsible
manner.
• The Indian Institute of Corporate Affairs (IICA) is registered as a society under
the Societies Registration Act of 1860 and works under the aegis of the Ministry of
Corporate Affairs. Hence statement 3 is correct.
• It is also a think tank that curates a repository of data and knowledge for
policymakers, regulators as well and other stakeholders working in the domain of
corporate affairs.

Source:
https://pib.gov.in/PressReleaseIframePage.aspx?PRID=1956690#:~:text=Mumbai%2C%2012
%20September%202023,Mumbai%20on%20September%2012%2C%202023

Q.61) Solution (b)

Statement Analysis:

• Skills Acquisition and Knowledge Awareness for Livelihood (SANKALP) is a centrally


sponsored scheme that aims to create convergence among all skill training activities.

https://t.me/visioniastestseries2024
https://upscmaterial.online/

IASBABA’S PRELIMS TEST SERIES (AIPTS) 2024 – TEST 10 (CURRENT AFFAIRS – SEPT ’23)

• Under a Central Sector Scheme, it is 100% funded by the Union government and
implemented by the Central Government machinery.
• Under a Centrally Sponsored Scheme (CSS) a certain percentage of the funding is
borne by the States in the ratio of 50:50, 70:30, 75:25, or 90:10 and the
implementation is by the State Governments. Hence statement 1 is incorrect.
• Skills Strengthening for Industrial Value Enhancement (STRIVE) is a centrally
sponsored scheme that aims to improve the relevance and efficiency of skills training
provided through Industrial Training Institutes (ITIs). Hence statement 2 is correct.

Source: https://pib.gov.in/PressReleaseIframePage.aspx?PRID=1957139

Q.62) Solution (c)

Statement Analysis:

The following initiatives are related to marine pollution:


• The UN Convention on the Law of the Sea aims to develop a legal framework
to prevent, reduce, and control pollution of the marine environment by dumping.
• London Convention aims to protect the marine environment from pollution caused
by the dumping of wastes and other matter into the ocean.
• The International Convention for the Prevention of Pollution from Ships aims
for the prevention of pollution of the marine environment by ships from operational
or accidental causes.
Hence option c is correct.

Source:
https://pib.gov.in/PressReleaseIframePage.aspx?PRID=1958029#:~:text=This%20initiative%2
0was%20announced%20by,Ho%20Chi%20Minh%2C%20and%20Jakarta

Q.63) Solution (a)

Statement Analysis:

• Deoxygenation in water bodies results in dead zones leading to mass fish kills and
other marine organism die-offs. Hence statement 1 is correct.
• Because deoxygenation leads to the reduction or depletion of dissolved oxygen
levels in aquatic environments. Hence statement 2 is correct.
• It can occur due to various natural and anthropogenic factors, disrupting the delicate
balance necessary for the survival of aquatic organisms.

https://t.me/visioniastestseries2024
https://upscmaterial.online/

IASBABA’S PRELIMS TEST SERIES (AIPTS) 2024 – TEST 10 (CURRENT AFFAIRS – SEPT ’23)

Source: https://www.downtoearth.org.in/news/climate-change/this-study-looked-at-how-
rivers-throughout-the-world-are-faring-and-the-findings-are-disturbing-91769

Q.64) Solution (c)

Statement Analysis:

• The Export Promotion Council for Handicrafts (EPCH) was established under the
Companies Act in the year 1986-87.
• It is a non-profit organisation, with an object to promote, support, protect, maintain,
and increase the export of handicrafts. Hence statement 1 is correct.
• It is an apex body of handicrafts exporters for the promotion of exports of handicrafts
from the country and projecting India's image abroad as a reliable supplier of high-
quality handicrafts.
• It has created the necessary infrastructure as well as marketing and information
facilities, which are availed both by the member exporters and importers. Hence
statement 2 is correct.

Source: https://pib.gov.in/PressReleseDetailm.aspx?PRID=1958243

Q.65) Solution (a)

Statement Analysis:

• Sickle Cell Disease (SCD) is a group of inherited red blood cell disorders with
symptoms such as chronic anaemia and delayed puberty. Hence statement 1 is
incorrect.
• In SCD, the red blood cells become hard and sticky and look like a C-shaped farm tool
called a “sickle”.
• It can be treated through blood transfusions, stem cell transplantation, and
hydroxyurea.
• Hydroxyurea is a medication that can help reduce the frequency of painful
episodes and prevent some of the long-term complications of the disease. Hence
statement 2 is correct.
• The National Sickle Cell Anemia Eradication Mission aims to eliminate sickle cell
anemia in India by 2047. Hence statement 3 is incorrect.

Source: https://www.thehindu.com/sci-tech/health/efforts-to-get-permanent-disability-
certificates-for-scd-patients-stuck-between-ministries-for-nearly-3-
years/article67299326.ece#:~:text=A%20plan%20to%20get%20permanent,get%20a%20mov
e%20on%20it

https://t.me/visioniastestseries2024
https://upscmaterial.online/

IASBABA’S PRELIMS TEST SERIES (AIPTS) 2024 – TEST 10 (CURRENT AFFAIRS – SEPT ’23)

Q.66) Solution (c)

Statement Analysis:

• Rubber requires deep and lateritic fertile soil with an acidic pH of 4.5 to 6.0. Hence
statement 1 is correct.
• It requires a tropical or subtropical climate with a minimum of about 1,200 mm per
year of rainfall and no frost.
• The minimum and maximum temperature should range from 25 to 34°C with 80
% relative humidity is ideal for cultivation.
• India is currently the world’s fifth-largest producer of natural rubber. Hence
statement 2 is incorrect.
• India is the second-largest consumer of natural rubber.
• Rubber is used in the manufacturing of tyres due to its excellent grip and wear
resistance, medical devices like gloves, syringe plungers, and equipment, sporting
goods like tennis balls, golf balls, and protective gear, and footwear for its cushioning
and slip-resistant properties. Hence statement 3 is correct.
• The Rubber Board is a statutory organization constituted under the Rubber Act of
1947. Hence statement 4 is correct.
• It functions under the administrative control of the Ministry of Commerce and
Industry. It is headed by a Chairman appointed by the Central Government and has
28 members representing various interests in the natural rubber industry.
• It is headquartered in Kottayam in Kerala.
• The Board is responsible for the development of the rubber industry in the country
by assisting and encouraging research, development, extension, and training
activities related to rubber.

Source: https://www.thehindu.com/business/rubber-board-to-increase-area-under-rubber-
in-
northeast/article67307179.ece#:~:text=In%20a%20move%20to%20expand,Sikkim%2C%20b
ut%20including%20West%20Bengal

Q.67) Solution (d)

Statement Analysis:

The planetary boundaries framework was first proposed by Johan Rockstrom to define
the environmental limits within which humanity can safely operate to maintain Earth's
stability and biodiversity. The nine planetary boundaries are:
• Ocean acidification
• Biogeochemical flows

https://t.me/visioniastestseries2024
https://upscmaterial.online/

IASBABA’S PRELIMS TEST SERIES (AIPTS) 2024 – TEST 10 (CURRENT AFFAIRS – SEPT ’23)

• Freshwater use
• Climate change
• Change in biosphere integrity
• Stratospheric ozone depletion.
• Land-system change
• Atmospheric aerosol loading
• Introduction of novel entities
Hence option d is correct.

Source: https://www.downtoearth.org.in/video/environment/six-of-9-planetary-boundaries-
have-been-breached-because-of-human-activities-study-
91818#:~:text=The%20six%20boundaries%20include%20climate,and%20novel%20entities%
20(consisting%20of

Q.68) Solution (b)

Statement Analysis:

The 7th report on antimicrobial use in animals released by the World Organisation for
Animal Health (WOAH) states:
• There is a 13% decrease in global antimicrobial usage in animals from 2017 to 2019.
Hence statement 1 is incorrect.
• Out of 80 countries, 49 in Asia, the Far East, Oceania, and Europe reported an overall
reduction in antimicrobial use.
• Around 68% of the participants have discontinued using antimicrobials as growth
promoters while 26% of participants continue to use growth promoters due to a lack
of proper legislation or regulations.
• The common antimicrobial growth promoters are flavomycin, bacitracin, avilamycin,
and tylosin. Hence statement 2 is correct.

Source: https://www.downtoearth.org.in/news/world/global-antibiotic-use-in-animals-dip-
by-13-in-3-years-but-regional-disparities-exist-
91801#:~:text=Published%3A%20Monday%2018%20September%202023&text=The%20glob
al%20usage%20of%20antimicrobials,on%20antibiotic%20use%20released%20recently

Q.69) Solution (a)

Statement Analysis:

• According to a recent study by the Journal Nature Communications, lion habitats


have seen a 60% decline in West and Central Africa. Hence statement 1 is correct.

https://t.me/visioniastestseries2024
https://upscmaterial.online/

IASBABA’S PRELIMS TEST SERIES (AIPTS) 2024 – TEST 10 (CURRENT AFFAIRS – SEPT ’23)

• It observed that southern African countries such as Botswana, South Africa,


Zimbabwe, and Namibia have recorded an increase in population by 12% between
1993 and 2014.
• It also states around 41.9% of the 62 geographical regions had less than 50 lions, 10
of them reported to have around 50-100 population of Lions and only seven
geographic locations across Africa were reported to have more than 1,000 lion
populations.
• Asiatic lions are slightly smaller than African lions and are listed as endangered on
the IUCN List. Hence statement 2 is incorrect.
• African lions are listed as vulnerable on the IUCN List.

Source: https://www.downtoearth.org.in/news/world/shrinking-lion-numbers-in-africa-40-
geographic-populations-have-50-or-fewer-big-cats-91802

Q.70) Solution (d)

Statement Analysis:

Narmada River is a west-flowing river of the peninsular region flowing through a rift valley
between the Vindhya Range on the north and the Satpura Range on the south. Its right
tributaries are Hiran, Tendori, Barna, Kolar, Man, Uri, Hatni, and Orsang while its left
tributaries are Burner, Banjar, Sher, Shakkar, Dudhi, Tawa, Ganjal, Chhota Tawa, Kundi, Goi,
and Karjan. It serves as a traditional boundary between North and South India. Its major
dams include Omkareshwar and Maheshwar. It originates from the Amarkantak peak of
Maikal Mountain and flows into the Gulf of Khambhat. It drains areas in Maharashtra,
Madhya Pradesh, and Gujarat. Hence option d is correct.

https://t.me/visioniastestseries2024
https://upscmaterial.online/

IASBABA’S PRELIMS TEST SERIES (AIPTS) 2024 – TEST 10 (CURRENT AFFAIRS – SEPT ’23)

Source: https://www.downtoearth.org.in/news/natural-disasters/swollen-narmada-
displaces-thousands-in-gujarat-was-this-an-avoidable-disaster--91807

Q.71) Solution (c)

Statement Analysis:

• Chennakeshava Temple is a 12th-century Hindu temple dedicated to Vishnu located


on the banks of the Yagachi River. Hence statement 1 is correct.

https://t.me/visioniastestseries2024
https://upscmaterial.online/

IASBABA’S PRELIMS TEST SERIES (AIPTS) 2024 – TEST 10 (CURRENT AFFAIRS – SEPT ’23)

• It was commissioned by King Vishnuvardhana in 1117 CE, on the banks of the Yagachi
River in Belur also called Velapura, an early Hoysala Empire capital.
• The richly sculptured exterior of the temple narrates scenes from the life of Vishnu
and his reincarnations and the epics, Ramayana, and Mahabharata. However, some
of the representations of Shiva are also included.
• Hoysaleswara temple is a 12th-century Hindu temple dedicated to Shiva located on
the banks of the Dwarasamudra tank. Hence statement 2 is correct.
• It was built in 1121 CE during the reign of the Hoysala King, Vishnuvardhana
Hoysaleshwara, and is most well-known for the more than 240 wall sculptures that
run all along the outer wall.

Source: https://indianexpress.com/article/india/karnatakas-sacred-ensembles-of-hoysalas-
inscribed-on-unesco-world-heritage-list-8945731/

Q.72) Solution (c)

Statement Analysis:

The Non-Aligned Movement was formed during the Cold War as an organization of States
that did not seek to formally align themselves with either the United States or the Soviet
Union but sought to remain independent or neutral. It was founded and held its first
conference (the Belgrade Conference) in 1961 under the leadership of Josip Broz Tito of
Yugoslavia, Gamal Abdel Nasser of Egypt, Jawaharlal Nehru of India, Kwame Nkrumah of
Ghana, and Sukarno of Indonesia.
The Relevance of the Non-Alignment Movement:
• It acted as a protector for small countries against Western hegemony as the third-
world countries were fighting against socio-economic problems since they had been
exploited for a long time by other developed nations. Hence statement 1 is correct.
• It has played an active role in preserving world peace. It aims to establish a peaceful
and prosperous world. It prohibited invasion of any country and promoted
disarmament and a sovereign world order. Hence statement 2 is correct.
• It acts as a bridge between the political and ideological differences existing in the
international environment by promoting an equitable world order. Hence statement
3 is correct.

Source: https://timesofindia.indiatimes.com/blogs/toi-editorials/no-bffs-needed/

Q.73) Solution (a)

Statement Analysis:

https://t.me/visioniastestseries2024
https://upscmaterial.online/

IASBABA’S PRELIMS TEST SERIES (AIPTS) 2024 – TEST 10 (CURRENT AFFAIRS – SEPT ’23)

• Food fortification began in India in the 1950s and it helps to combat hidden hunger.
Hence statement 1 is correct.
• Hidden hunger occurs when people get enough food to eat, but the food lacks
essential nutrients, vitamins, and minerals, leading to malnutrition. This condition is
often concealed until noticeable health issues arise, such as impaired cognitive
abilities and birth defects.
• Because it adds essential vitamins and minerals to increase the nutritional value of
common foods. Hence statement 2 is correct.
• The Food Safety and Standards Authority of India established fortification standards
in 2016 and introduced the ‘+F’ logo to facilitate food producers in joining the
fortification efforts. This helps in enhancing the nutritional value of food items and
improving the overall health of the population.

Source: https://www.livemint.com/opinion/online-views/rice-fortification-can-help-tackle-
our-problem-of-hidden-hunger-11695228502682.html

Q.74) Solution (b)

Statement Analysis:

• Battery energy storage systems (BESS) are smart systems as they


use algorithms to interact with the grid and make decisions regarding storing and
releasing surplus energy. They primarily utilize lithium-ion batteries due to their
characteristics like high energy density, decreasing costs, and extended lifespan.
• Battery energy storage systems (BESS) preserve energy generated from renewable
sources like sunlight and wind. It balances the supply and demand of renewable
energy by releasing it into the grid when required. Hence statement 1 is incorrect.
• Renewable energy sources have intermittent generation patterns because solar
energy is available during the day and heavy wind is also a seasonal phenomenon. It
can cause abrupt surges or declines in power generation, leading to power outages.
It overcomes the issue of intermittent generation patterns of renewable energy
sources by enhancing grid stability. Hence statement 2 is correct.

Source: https://www.livemint.com/news/india/all-charged-up-india-s-battery-storage-plans-
11695234124822.html

Q.75) Solution (d)

Statement Analysis:

https://t.me/visioniastestseries2024
https://upscmaterial.online/

IASBABA’S PRELIMS TEST SERIES (AIPTS) 2024 – TEST 10 (CURRENT AFFAIRS – SEPT ’23)

Disease Plant

1. Red Rot Sugarcane

2. Pink Bollworm Cotton

3. Canker Tomato

4. Downy Mildew Grapes, Cabbage,


Cauliflower, Bajra,
Mustard
5. Leaf Roll Potato

6. Bunchy Top Banana

7. Mosaic Tobacco

8. Leaf Curl Papaya

9. Red Leaf Carrot

10. White Rust Raddish, Turnip

11. Rust Coffee

12. Powdery Mildew Wheat

13. Foot Rot Paddy, Papaya

14. Blast Rice

15. Tikka Groundnut

16. Wilt Pigeon Pea, Cotton

17. Ergot Bajra

18. Late Blight Potato

19. Green Ear Bajra

https://t.me/visioniastestseries2024
https://upscmaterial.online/

IASBABA’S PRELIMS TEST SERIES (AIPTS) 2024 – TEST 10 (CURRENT AFFAIRS – SEPT ’23)

Hence option d is correct.

Source: https://indianexpress.com/article/india/rajasthan-haryana-pest-havoc-cotton-fields-
8960994/

Q.76) Solution (a)

Statement Analysis:

World University Rankings 2024 was founded in 2004 and is published by the Times Higher
Education. The Times Higher Education World University Rankings 2024 include 1,904
universities across 108 countries and regions. These rankings are the most comprehensive,
rigorous, and balanced global rankings. It covers their core missions of teaching, research,
knowledge transfers, and internationalization.
The key findings of the Times Higher Education’s World University Rankings 2024:
• The US (169 institutions) is the most-represented country while India (91
institutions)is the fourth most-represented country in terms of institutions. Hence
statement 1 is correct.
• The Indian Institute of Science, Bangalore is the top university in India while the
University of Oxford tops the overall ranking list.
• The next highest-ranked universities in India are Anna University, Jamia Millia
Islamia, Mahatma Gandhi University, and Shoolini University of Biotechnology and
Management Sciences. Hence statement 2 is incorrect.

Source: https://indianexpress.com/article/education/the-rankings-2024-iisc-ranked-best-in-
india-oxford-university-best-in-world-8959549/

Q.77) Solution (c)

Statement Analysis:

CALIPSO Mission was launched in 2006 to enhance our understanding of clouds and
aerosols and their roles in Earth’s climate system. It was jointly launched by France and the
U.S. to the sun-synchronous orbit. It aims to provide valuable data for climate
change research by monitoring key atmospheric parameters and contributes to improved air
quality assessments and weather forecasting by providing real-time atmospheric data.
Hence option c is correct.

Source: https://indianexpress.com/article/technology/science/nasa-calipso-mission-
blasting-lasers-earth-8959943/

https://t.me/visioniastestseries2024
https://upscmaterial.online/

IASBABA’S PRELIMS TEST SERIES (AIPTS) 2024 – TEST 10 (CURRENT AFFAIRS – SEPT ’23)

Q.78) Solution (b)

Statement Analysis:

• The Battle of Haifa was fought during the First World War and is commemorated as
Hafia Day on 23rd September, since 2010 by India and Israel. Hence statement 1 is
incorrect.
• It commemorates the Hafia capture following a daring cavalry action by the 15th
Imperial Service Cavalry Brigade during World War I.
• It involved the British Empire fighting against the Ottoman Kingdom, the German
Empire, and Austria-Hungary. Hence statement 2 is correct.
• The Battle of Haifa is a testament to the valour of Indian soldiers, including Sikhs and
Rajputs, in global conflicts. It highlights India’s contributions during the First World
War.

Note: Teen Murti Chowk in India was renamed Teen Murti Haifa Chowk in honour of the
Battle of Haifa’s centenary.

Source: https://indianexpress.com/article/explained/explained-history/battle-of-haifa-
8952515/

Q.79) Solution (b)

Statement Analysis:

• The Dadasaheb Phalke Lifetime Achievement Award is India’s highest cinema


honour presented at the National Film Awards ceremony. Hence statement 1 is
correct.

https://t.me/visioniastestseries2024
https://upscmaterial.online/

IASBABA’S PRELIMS TEST SERIES (AIPTS) 2024 – TEST 10 (CURRENT AFFAIRS – SEPT ’23)

• Dadasaheb Phalke was the director of India’s inaugural full-length feature film, “Raja
Harishchandra,” in 1913. He is known as the “Father of Indian Cinema”.
• Devika Rani is the first awardee of this award which was initiated in 1969. Hence
statement 2 is correct.
• Waheeda Rehman will be honoured with the 53rd Dadasaheb Phalke Lifetime
Achievement Award for the year 2023.
• It is presented annually at the National Film Awards ceremony by the Directorate of
Film Festivals.
• Its prize includes a Swarna Kamal (Golden Lotus) medallion, a shawl, and a cash prize
of 10 lakh. Hence statement 3 is incorrect.

Source:
https://pib.gov.in/PressReleaseIframePage.aspx?PRID=1960823#:~:text=Waheeda%20Rehm
an%20to%20be%20honoured%20with%2053rd%20Dadasaheb%20Phalke%20Lifetime%20Ac
hievement%20Award&text=Union%20Minister%20Shri%20Anurag%20Thakur,Award%20for
%20the%20year%202021

Q.80) Solution (c)

Statement Analysis:

• The Central Board of Indirect Taxes and Customs (CBIC) is a part of the Department
of Revenue under the Ministry of Finance. Hence statement 1 is correct.
• It is headquartered in New Delhi.
• It is a statutory body established under the Central Boards of Revenue Act, of 1963.
Hence statement 2 is correct.
• It was formed in 1964 when the Central Board of Revenue was split into the Central
Board of Direct Taxes (CBDT) and the Central Board of Excise and Customs.
• The Central Board of Excise and Customs was renamed the Central Board of Indirect
Taxes and Customs in 2018.
• It is the nodal national agency responsible for administering customs, central excise,
service tax, GST, and narcotics in India. Hence statement 3 is correct.
• It is the administrative authority for its subordinate organizations, including Custom
Houses, Central Excise and Central GST Commissionerate, and the Central Revenues
Control Laboratory.

Source: https://newsonair.gov.in/News?title=CBIC-plays-crucial-role-in-fight-against-illicit-
trade%3a-CBIC-Chairman-Sanjay-Kumar-Agarwal&id=468533

https://t.me/visioniastestseries2024
https://upscmaterial.online/

IASBABA’S PRELIMS TEST SERIES (AIPTS) 2024 – TEST 10 (CURRENT AFFAIRS – SEPT ’23)

Q.81) Solution (b)

Statement Analysis:

• The Global Innovation Index 2023 is published annually by the World Intellectual
Property Organization (WIPO). Hence statement 1 is correct.
• It is a leading reference for measuring an economy’s innovation ecosystem
performance.
• It is also a valuable benchmarking tool used by policymakers, business leaders, and
other stakeholders to assess progress in innovation over time.
• WIPO is a self-funding agency of the United Nations, with 193 member states.
• Switzerland is the most innovative economy followed by Sweden, the United States,
and the United Kingdom. Hence statement 2 is correct.
• India retained the 40th rank out of 132 economies. Over the past few years, India
has consistently climbed the ranks in the GII, rising from 81st place in 2015 to its
current position.
• Tokyo–Yokohama is the top science and technology innovation cluster in the world.
It is followed by Shenzhen–Hong Kong–Guangzhou, Seoul, Beijing, and Shanghai-
Suzhou. Hence statement 3 is incorrect.

Source: https://newsonair.gov.in/News?title=India-retains-40th-rank-in--Global-Innovation-
Index-2023&id=468514

Q.82) Solution (c)

Statement Analysis:

• The Psyche mission is a NASA mission to study a metal-rich asteroid named Psyche.
Hence statement 1 is incorrect.
• This is NASA’s first mission to study an asteroid that has more metal than rock or ice.
• It is powered by solar electric propulsion and has a magnetometer, a gamma-ray, a
multispectral imager, and a neutron spectrometer. Hence statement 2 is correct.
• The spacecraft will start sending images to Earth as soon as it spots the asteroid.
• It will be launched from NASA’s Kennedy Space Center, Florida, USA by SpaceX Falcon
Heavy launch vehicle.
• The mission helps to understand the early solar system and the formation of
terrestrial planets. Hence statement 3 is correct.
• Asteroid Psyche is located in the main asteroid belt between Mars and Jupiter.
• It is believed to be a remnant core of a planetesimal, composed primarily of iron-
nickel metal. Hence statement 4 is correct.

https://t.me/visioniastestseries2024
https://upscmaterial.online/

IASBABA’S PRELIMS TEST SERIES (AIPTS) 2024 – TEST 10 (CURRENT AFFAIRS – SEPT ’23)

Image: Graphicnews
Source: https://www.indiatoday.in/science/story/nasa-to-launch-psyche-mission-to-metal-
rich-asteroid-to-find-how-earth-formed-2440170-2023-09-25

Q.83) Solution (a)


Statement Analysis:
• Cabomba furcata is a species of aquatic plant in the water shield family. It is known
by the common names red cabomba and forked fanwort.
• It is native to Central and South America and as far north as Cuba and the tip of
Florida.
• It reaches a maximum height between 30 and 80 centimeters and is up to 8
centimeters wide. It bears purple flowers and is used as an aquarium plant.
• Cabomba furcata is economically and ecologically hindering the growth of native
aquatic plants and freshwater fish in Kerala. Hence statement 1 is correct.
• Its active stem propagation prevents light from penetrating the surface of water and
it requires a huge quantity of oxygen, resulting in a decline in biodiversity. Hence
statement 2 is correct.

Source: https://www.thehindu.com/news/national/kerala/kole-wetlands-face-threat-of-
alien-plants/article67341793.ece

https://t.me/visioniastestseries2024
https://upscmaterial.online/

IASBABA’S PRELIMS TEST SERIES (AIPTS) 2024 – TEST 10 (CURRENT AFFAIRS – SEPT ’23)

Q.84) Solution (b)


Statement Analysis:
• Valmiki TIGER RESERVE IS THE ONLY TIGER RESERVE IN BIHAR. Hence statement 1
is correct.
• Madhya Pradesh has the highest number of tiger reserves with a count of seven.
Karnataka has five tiger reserves. Hence statement 2 is incorrect.
• NAGARJUNSAGAR-SRISAILAM TIGER RESERVE is the largest tiger reserve in India.
• Kamlang Tiger Reserve is the second-largest tiger reserve in India. Hence statement 3
is incorrect.
• Bhor Tiger Reserve is the smallest tiger reserve in India. Hence statement 4 is
correct.

Source: https://ntca.gov.in/tiger-reserves/#tiger-reserves-2
https://www.downtoearth.org.in/news/wildlife-biodiversity/bihar-to-get-its-second-tiger-
reserve-in-kaimur-soon-91986

Q.85) Solution (d)


Statement Analysis:

• Nilgiri tahr is the state animal of Tamil Nadu. Hence statement 1 is correct.
• The adult males of Nilgiri Tahr species develop a light grey area or “saddle” on their
backs and are hence called “Saddlebacks”.
• It is the only mountain ungulate in southern India. Hence statement 2 is correct.
• It is endemic to the Nilgiri Hills and the southern part of the Western Ghats in the
states of Tamil Nadu and Kerala. Hence statement 3 is correct.
• It is found in the open montane grassland habitat of the South Western Ghats
montane rain forests eco-region.
• Eravikulam National Park is home to the largest population of Nilgiri Tahr.
• It is protected under Schedule I of the Wildlife Protection Act of 1972. Hence
statement 4 is correct.
• It is listed as endangered on the IUCN List.

Source: https://www.thehindu.com/sci-tech/energy-and-environment/tamil-nadu-kerala-to-
join-hands-to-count-endangered-nilgiri-tahr/article67338667.ece

Q.86) Solution (a)

https://t.me/visioniastestseries2024
https://upscmaterial.online/

IASBABA’S PRELIMS TEST SERIES (AIPTS) 2024 – TEST 10 (CURRENT AFFAIRS – SEPT ’23)

Statement Analysis:

Form of Painting Work


1. Malwa Painting Rasikapriya
2. Mewar Painting Ragamala
3. Bundi Painting Bhairavi Ragini
Hence option a is correct.

• Malwa School of painting flourished between 1600 and 1700 CE and is most
representative of the Hindu Rajput courts. It shows a fondness for rigorously
flat compositions, black and chocolate-brown backgrounds, figures shown against a
solid colour patch, and architecture painted in lively colour. Its most appealing
features are a primitive charm and a simple childlike vision.
• Mewar painting is one of the most important schools of Indian miniature painting of
the 17th and 18th centuries. It is a school in the Rajasthani style and was developed
in the Hindu principality of Mewar (in Rajasthan state). It is characterized by simple
bright colour and direct emotional appeal.
• Bundi school of painting is dated back to 1625 AD. It is characterised by the rich and
glowing colours, the rising sun in golden colour, crimson-red horizon, overlapping
and semi-naturalistic trees.

Source: https://theprint.in/feature/around-town/mewar-style-mahabharata-painted-by-
muslim-what-2-art-enthusiasts-found-buried-in-udaipur-fort/1777075/?amp

Q.87) Solution (b)


Statement Analysis:

• Mushkoh Valley is known as the valley of wild tulips as the meadows of Mushkoh
offer boisterous wild tulip flowers. Hence statement 1 is incorrect.
• The valley is also home to the endangered Himalayan yew.
• Burzil Pass connects the Kashmir valley with the Deosai plains of Ladakh. It is situated
at an altitude of 4100 meters above sea level. Hence statement 2 is correct.
• Kaobal Gali connects Gurez Valley to Mushkoh Valley. It is the highest pass at a
height of 4,167 meters in Gurez. Hence statement 3 is correct.

https://t.me/visioniastestseries2024
https://upscmaterial.online/

IASBABA’S PRELIMS TEST SERIES (AIPTS) 2024 – TEST 10 (CURRENT AFFAIRS – SEPT ’23)

Source: https://www.thehindu.com/news/national/kaobal-gali-mushkoh-valley-the-
battlefields-of-kargil-war-opens-up-for-tourists/article67338412.ece/amp/

Q.88) Solution (b)


Statement Analysis:

• The India Post Payments Bank Limited (IPPB) accepts deposits and offers remittance
services, mobile banking, and third-party fund transfers. Hence statement 1 is
correct.
• The IPPB was launched in 2018 and is governed by the Reserve Bank of India (RBI).
• It is a payments bank of the Indian postal department.
• It works through a network of post offices and nearly 4 lakh postmen.
• Its objective is to build the most accessible, affordable, and trusted bank for the
common man in India.
• It offers 3 types of saving accounts - Regular Account – Safal, Basic Savings Bank
Deposit Account (BSBDA) – Sugam, and BSBDA Small – Saral. Hence statement 2 is
correct.
• They offer a 4% interest rate on savings accounts.
• The maximum limit on deposits for current and savings accounts is Rs 1 lakh and
they can issue debit cards and ATM cards. Hence statement 3 is incorrect.
• They cannot issue credit cards and cannot loan money.

Source: https://timesofindia.indiatimes.com/education/news/ippb-executive-admit-card-
2023-released-on-ippbonline-com-download-here/articleshow/103957120.cms

Q.89) Solution (b)


Statement Analysis:

• The Unified Registration Portal for GOBARdhan is launched by the Ministry of Jal
Shakti. Hence statement 1 is incorrect.
• Anyone who operates or intends to set up a biogas/ CBG/ Bio CNG plant in
India can obtain a registration number by registering in this unified registration
portal. The registration number is required to avail of benefits/ support from other
Ministries/ Departments.
• It aims to streamline the registration of compressed bio-gas and biogas plants
nationwide. Hence statement 2 is correct.
• It acts as a data repository for different activities of Gobardhan.

https://t.me/visioniastestseries2024
https://upscmaterial.online/

IASBABA’S PRELIMS TEST SERIES (AIPTS) 2024 – TEST 10 (CURRENT AFFAIRS – SEPT ’23)

• It provides real-time tracking, ensuring transparency, and social accountability of


the scheme. Hence statement 3 is correct.

Source: https://newsonair.gov.in/News?title=Unified-Registration-Portal-for-GOBARdhan-
introduced-to-streamline-registration-of-Compressed-Bio-Gas-and-biogas-plants-
nationwide&id=468354

Q.90) Solution (c)


Statement Analysis:

Rhino Species IUCN Status


1. Javan Rhinos Critically Endangered
2. Sumatran Rhinos Critically Endangered
3. Greater One-Horned Rhinos Vulnerable
4. White Rhinos Near Threatened

Hence option c is correct.

https://t.me/visioniastestseries2024
https://upscmaterial.online/

IASBABA’S PRELIMS TEST SERIES (AIPTS) 2024 – TEST 10 (CURRENT AFFAIRS – SEPT ’23)

Note: The World Rhino Day is observed on 22nd September every year. Its objective is to
spread awareness for all five species of rhino and the work being done to save them. It was
first announced by WWF-South Africa in 2010. This special day provides the opportunity for
cause-related organizations, NGOs, zoos, and members of the public to celebrate rhinos in
their own unique ways.

Source: https://www.downtoearth.org.in/news/world/run-up-to-world-rhino-day-2023-
poaching-climate-change-habitat-loss-threaten-global-population-91878

Q.91) Solution (c)


Statement Analysis:

• India will be included in the Government Bond Index-Emerging Markets (GBI-EM)


published by JP Morgan Chase & Co from June 2024. Hence statement 1 is correct.
• The index comprises government bonds issued by emerging market
countries. These countries can differ significantly in terms of their economic
development, creditworthiness, and the size of their bond markets.
• Investment decisions and portfolio allocations in the emerging market debt space
are often guided by this index.
• It acts as a push factor to prompt foreign inflows into India and it brings down the
cost of borrowing for the government. Hence statement 2 is incorrect.
• It will support the Indian rupee and bond markets, and improve the country’s credit
rating.

Source: https://indianexpress.com/article/business/economy/jp-morgan-india-emerging-
markets-bond-index-8951000/

Q.92) Solution (a)


Statement Analysis:

• Kallanai Dam in Tamil Nadu is the oldest dam in India. Hence statement 1 is correct.
• Tehri Dam in Uttarakhand is the tallest dam in India. Hence statement 2 is correct.
• Hirakud Dam in Odisha is the longest dam in India. Hence statement 3 is correct.

https://t.me/visioniastestseries2024
https://upscmaterial.online/

IASBABA’S PRELIMS TEST SERIES (AIPTS) 2024 – TEST 10 (CURRENT AFFAIRS – SEPT ’23)

Source: https://www.thehindu.com/news/national/other-states/odisha-announces-plan-to-
rehabilitate-those-displaced-by-construction-of-the-hirakud-dam/article67263445.ece

Q.93) Solution (d)


Statement Analysis:

• Hormones produced by the adrenal cortex include cortisol and androgens. Hence
statement 1 is correct.
• Cortisol is the primary hormone produced by the adrenal cortex and plays a vital role
in regulating metabolism, immune response, and the body’s response to stress.
• Androgens play a role in the development of male sexual characteristics and
contribute to the sex drive in both men and women.
• Hormones produced by the thyroid gland include calcitonin and triiodothyronine.
Hence statement 2 is correct.

https://t.me/visioniastestseries2024
https://upscmaterial.online/

IASBABA’S PRELIMS TEST SERIES (AIPTS) 2024 – TEST 10 (CURRENT AFFAIRS – SEPT ’23)

• Calcitonin helps regulate calcium levels in the body by reducing the amount of
calcium released from bones.
• Triiodothyronine is the more active form of thyroid hormone and is responsible for
regulating metabolic processes such as energy production, heart rate, and body
temperature.
• Hormones produced by the endocrine pancreas include somatostatin and insulin.
Hence statement 3 is correct.
• Somatostatin is a hormone that inhibits the release of both insulin and glucagon,
helping to regulate blood sugar levels.
• nsulin is a hormone that regulates the body’s blood sugar levels. It is produced by
beta cells in the islets of Langerhans and helps to lower blood sugar by promoting
the uptake and use of glucose by cells throughout the body.
• Hormones produced by the gonads include testosterone and estrogen. Hence
statement 4 is correct.
• Testosterone is produced by the Leydig cells in the testes and is responsible for the
development of male reproductive organs, secondary sexual characteristics, and the
regulation of sperm production.
• Estrogen is produced by the follicles in the ovaries and is responsible for the
development of female reproductive organs, secondary sexual characteristics, and
the regulation of the menstrual cycle.

Source: https://www.thehindu.com/sci-tech/health/pcos-hidden-toll-of-academic-pressure-
on-our-daughters-health/article67327587.ece

Q.94) Solution (d)


Statement Analysis:
All India Financial Institutions (AIFIs) is a group composed of financial regulatory bodies
that play a pivotal role in the financial markets by assisting the proper allocation of
resources, sourcing from businesses that have a surplus and distributing to others who have
deficits. They act as an intermediary between borrowers and final lenders, providing safety
and liquidity.
The following AIFIs in India are regulated by the Reserve Bank of India:
• National Bank for Financing Infrastructure and Development (NaBFID)
• National Housing Bank (NHB)
• Small Industries Development Bank of India (SIDBI)
• Export-Import Bank of India (EXIM Bank)
• National Bank for Agriculture and Rural Development (NABARD)
Hence option d is correct.

https://t.me/visioniastestseries2024
https://upscmaterial.online/

IASBABA’S PRELIMS TEST SERIES (AIPTS) 2024 – TEST 10 (CURRENT AFFAIRS – SEPT ’23)

Source: https://www.thehindubusinessline.com/money-and-banking/rbi-introduces-basel-
iii-capital-framework-for-all-india-financial-institutions/article67331511.ece

Q.95) Solution (b)


Statement Analysis:

• The Indian Skimmer is listed as endangered on the IUCN List. Hence statement 1 is
incorrect.
• It is an unusual-looking bird with a striking red, orange beak where the lower bill is
longer than the upper bill.
• Globally found in the coastal estuaries of western and eastern India, and Bangladesh.
• It feeds on fish, larvae, insects, and shrimps.
• It favours slow-moving rivers with sandbar habitats formed seasonally during
summers, around lakes and adjacent marshes, estuaries, and coasts.
• It breeds between February to June and raise one to three chicks per clutch. Hence
statement 2 is correct.
• It is protected under Schedule I of the Wildlife Protection Act of 1972. Hence
statement 3 is correct.

Source: https://timesofindia.indiatimes.com/city/lucknow/endangered-indian-skimmer-
spotted-at-dudhwa-tiger-reserve/articleshow/103843596.cms?from=mdr

Q.96) Solution (c)


Statement Analysis:
Five Eyes Alliance was established Post-World War II. The term “Five Eyes” refers to the five
countries’ collective efforts to gather and share signals intelligence (SIGINT) to address
common security threats and challenges. Its members are Australia, Canada, New Zealand,
the United Kingdom, and the United States. Hence option c is correct.

Source: https://indianexpress.com/article/explained/everyday-explainers/india-canada-five-
eyes-alliance-intelligence-explained-8951916/

Q.97) Solution (c)


Statement Analysis:

https://t.me/visioniastestseries2024
https://upscmaterial.online/

IASBABA’S PRELIMS TEST SERIES (AIPTS) 2024 – TEST 10 (CURRENT AFFAIRS – SEPT ’23)

• An artificial reef is a manmade structure that may mimic some of the characteristics
of a natural reef. Hence statement 1 is correct.
• They are placed in areas where there is little bottom topography or near coral reefs
to attract marine populations.
• They serve to protect coral reefs from human-induced damages as well as
supporting biodiversity and healthy ecosystems.
• Submerged shipwrecks, oil and gas platforms, bridges, and lighthouses function as
artificial reefs. Hence statement 2 is correct.
• Materials used to construct artificial reefs have included rocks, cinder blocks, and
even wood and old tires.

Source: https://pib.gov.in/PressReleasePage.aspx?PRID=1959725

Q.98) Solution (b)


Statement Analysis:

• The World Federation of the Deaf (WFD) is an international non-profit and non-
governmental organization of deaf associations. Hence statement 1 is correct.
• Its objective is to promote the human rights of deaf people in accordance with the
principles and objectives of the United Nations Charter, Universal Declaration of
Human Rights, UN Convention on the Rights of Persons with Disabilities (CRPD), 2030
Agenda and Sustainable Development Goals (SDGs), and other Human Rights
Treaties.
• It is headquartered in Helsinki, Finland.
• The UN General Assembly has proclaimed 23rd September as the International Day
of Sign Languages. Hence statement 2 is incorrect.
• It commemorates the date that the World Federation of the Deaf (WFD) was
established in 1951.
• The Theme for the International Day of Sign Languages 2023 is “A World where Deaf
People Everywhere can Sign Anywhere!”. Hence statement 3 is correct.

Source: https://www.pib.gov.in/PressReleasePage.aspx?PRID=1959726

Q.99) Solution (c)


Statement Analysis:

https://t.me/visioniastestseries2024
https://upscmaterial.online/

IASBABA’S PRELIMS TEST SERIES (AIPTS) 2024 – TEST 10 (CURRENT AFFAIRS – SEPT ’23)

• Greenwishing refers to organisations expressing a desire to be more environmentally


responsible without taking concrete actions to achieve those goals. Hence statement
1 is correct.
• It’s like making a wish for sustainability without any tangible actions directed in the
required direction.
• Greenwashing refers to the deceptive practice of making false or exaggerated claims
about the environmental friendliness of a company’s products, services, or practices.
Hence statement 2 is correct.
• Greenhushing implies a situation where an organisation intentionally downplays
their positive environmental achievements. Hence statement 3 is correct.
• It might involve not publicising sustainable practices for various reasons, such as
modesty, fear of criticism, or reducing external communication.

Source: https://www.livemint.com/opinion/online-views/climate-finance-must-get-beyond-
greenwishing-and-greenwashing-11695306060499.html

Q.100) Solution (b)


Statement Analysis:

• The National Medical Commission (NMC) has replaced the Medical Council of India
(MCI) constituted under the Indian Medical Council Act, 1956. Hence statement 1 is
correct.
• It is a statutory body under the National Medical Commission Act, 2019. Hence
statement 2 is correct.
• It regulates medical education and medical professionals and grants recognition of
medical qualifications gives accreditation to medical schools, grants registration to
medical practitioners, monitors medical practice, and assesses the medical
infrastructure in India.
• Under-Graduate Medical Education Board is one among the four autonomous
boards under NMC. Hence statement 3 is incorrect.
• It consists of 33 members including Chairman (medical professionals only), 10 ex-
officio members and 22 parttime members.
• The other three autonomous boards under NMC are Post-Graduate Medical
Education Board, Medical Assessment and Rating Board, and Ethics and Medical
Registration Board.
Source: https://newsonair.gov.in/News?title=National-Medical-Commission-gets-global-
recognition%2c-paving-way-for-Indian-medical-graduates-to-practice-abroad&id=468156

https://t.me/visioniastestseries2024

You might also like